Neuro 3 Test Yourself

Réussis tes devoirs et examens dès maintenant avec Quizwiz!

A 76-year-old woman had sudden onset of paralysis and sensory loss of the left foot and leg, sparing the upper extremities. A) Anterior cerebral artery syndrome B) Basilar artery syndrome C) Middle cerebral artery syndrome D) Posterior cerebral artery syndrome E) Posterior inferior cerebellar artery syndrome

(A) Anterior cerebral artery syndrome, based on the finding of foot and leg weakness suggesting disease in the medial surface of the frontal lobe, sparing the upper extremities, which are represented in the lateral surfaces of the frontal lobe.

A 47-year-old married mother of two was neuropsychiatrically well until two weeks prior to clinic examination, when she developed right eye pain. At first, the pain was moderate and occurred intermittently, and then it became intense and continuous. It didn't throb or pulsate. She had no trouble seeing, photosensitivity, aura, nausea, vomiting, scotomata, rash, or tearing. When she looked in the mirror, neither eye was red. A week before the clinic visit, she had three episodes, two days apart, in which "I couldn't get my bearings." The first two involved "the stacks of materials on my desk. I knew what the materials were, but couldn't figure out their relationship to each other. This was weird because I'm well-organized." A similar episode occurred at a department party when, while dancing, "I couldn't decide whether to move my left or right foot." After the episodes were over, her spatial orientation returned to normal. She smokes a pack a day. She had left leg phlebitis after her second childbirth. She took birth control pills when she was in her 20s, but currently takes no medications. She has no history of seizures or diabetes, heart disease or head trauma. She has an identical twin sister with no history of facial pain, transient spatial disorientation, vascular disease, diabetes or seizures. A thorough physical, including behavioral brain and general neurologic examination revealed no neurologic, general medical or psychiatric abnormalities. 25. Applying the principle of parsimony, the most likely explanation of her symptoms is A) Cluster headache due to an autoimmune phenomenon B) Atypical migraine without nausea, vomiting or scotomata C) Right middle cerebral artery saccular aneurysm D) Simple partial seizures affecting the left parietal lobe E) Astrocytoma of the right occipital lobe

. (C) Right middle cerebral artery saccular aneurysm is a parsimonious and correct response that explains all the findings. The middle cerebral artery is adjacent to the third nerve, which supplies pain fibers to the meninges and intracranial vessels. The right middle cerebral artery also supplies the non-dominant parietal lobe which handles spatial orientation, and the frontal lobe, which handles planning. Cluster headache also produces severe pain, usually localized in and around the eye, often with associated with blocked nostrils, rhinorrhea, injected conjunctiva, lacrimation, miosis and flushing or edema in the back of the cheek, all lasting 15-180 minutes. The patient has severe eye pain, but none of the other symptoms. Migraine is usually unilateral and pulsating and does not center in the eye. Eye pain, spatial disorientation and impaired spatially-oriented planning are rarely if ever signs of simple partial seizures. Occipital lobe astrocytomas and other occipital tumors often cause occipital area pain, not eye pain. They can cause increased intracranial pressure with nausea, vomiting and headache, none of which the patient has. They can cause visual field and other visual abnormalities, which this patient does not have. They don't tend to impede spatial orientation or visual-spatial planning.

A 60-year-old right-handed man was neuropsychiatrically well until his wife noticed that suddenly, his speech slurred and one corner of his mouth drooped. His left naso-labial fold is more clearly defined than the right one. He can furrow both sides of his forehead and close his eyes tightly, symmetrically and bilaterally. He has right ankle clonus. Mini-mental State score is 26/30. Of the following, which is he most likely to have? A) Absence seizures B) Left ankle clonus C) Reduced vibratory sense on the right D) Bi-temporal hemianopia E) Right Babinski sign

. (E) Right Babinski sign goes along with right ankle clonus, reflecting right corticospinal tract pathology. Absence seizures start in childhood, are characterized by unconsciousness lasting a few seconds, with resumption of the person's pre-seizure activity immediately afterwards. There is no association of absence seizures with long tract (e.g., corticospinal, spinothalamic) signs. Left ankle clonus occurs only with left corticospinal or right motor cortex disease, for which there is no evidence in this case. Reduced vibratory sense occurs with dorsal column disease, for which there is no evidence here. Bitemporal hemianopia results from optic chiasm disease, for which there is no evidence here.

A 70-year-old married man was neuropsychiatrically well until six years ago, when he started having tremors, gait problems and falling, which have increased gradually and progressively. Four months ago, he started having a sad mood with pessimism, apathy, trouble falling asleep, and occasional suicidal ideation without suicidal intent or plans. Recently, he became unable to play golf. He has no past or family history of psychiatric illness. He is alert, has an expressionless face, a slow gait with reduced arm swing, and muscle rigidity. He has a bilateral tremor, with his thumbs rhythmically apposing his forefingers, most prominent when his hands rest at his sides. His mood is sad, but it improves as the interview progresses. He has mild problems with recent and remote memory, and his memory improves with cuing. His Mini-mental State score is 24/30. 5. Of the following, the most likely diagnosis is A) Mild neurocognitive disorder due to Parkinson's disease (Parkinson's disease) B) Mild neurocognitive disorder due to Alzheimer's disease (Alzheimer's disease) C) Mild neurocognitive disorder due to normal-pressure hydrocephalus (Normal-pressure hydrocephalus) D) Persistent depressive disorder (Dysthymic disorder) E) Bipolar II disorder

5. (A) Parkinson's disease, characterized in this case by onset at 63, inability to perform his usual skilled motor movements, pill-rolling resting tremor, rigidity, slow gait with reduced arm swing, expressionless face, cognitive dysfunction and atypical depression. Alzheimer's disease is characterized by late age of onset and temporoparietal dysfunction without movement disorder. Normal-pressure hydrocephalus is characterized by dementia with frontal signs, gait abnormality and urinary incontinence. Persistent depressive disorder (Dysthymic disorder), a chronic, usually mild depression that occurs on most days of the week, most weeks of the month and most months of the year, without any two-month period of wellness, has an earlier age of onset with no movement or cognitive disorder. Bipolar II disorder is characterized by "typical" major depressive episodes and one or more hypomanic episodes. Tremor does not typify bipolar disorder, except when it is a sign of lithium or valproic acid toxicity.

Doctor: Tell me what you see in this picture. Patient: Tell me what is in the picture. Doctor: Repeat after me, say 'no ifs ands or buts.' Patient: Repeat after me, say no ifs ands or buts. Doctor: What is in the picture? Patient: In the picture it's too early to tell how many gave them that. Doctor: Tell me, what's happening in the picture? Patient: In the picture nine or ten houses are too old for them. A) Broca's aphasia B) Catatonia C) Circumstantiality D) Extrasylvian sensory aphasia E) Normal language, speech and cognitive function, no abnormality present F) Spatial neglect G) Wernicke's aphasia

(A) Broca's aphasia, characterized by dysarthric, telegraphic speech with impaired repetition, reasonably good comprehension (except for sentences with many functor words), and no paraphasic speech.

10. A 76-year-old hypertensive man has sudden onset of delusions of persecution, left hemiparesis and left homonymous hemianopia. He has no past or family history of psychiatric illness. His alertness is reduced and his pulse is irregularly irregular. Of the following, the most likely immediate cause of his persecutory delusions is A) Cerebral embolus B) Alzheimer's disease C) Delusional disorder D) Schizophrenia, paranoid type E) A degenerative disease of his basal ganglia

(A) Cerebral embolus best explains the sudden onset of neuropsychiatric symptoms and signs in the presence of atrial fibrillation, suggested by the irregularly irregular heartbeat. Alzheimer's disease develops gradually over months and years. Neither delusional disorder nor schizophrenia nor basal ganglia disease is associated with long tract signs or cardiac irregularities.

A 54-year-old man was neuropsychiatrically well until four months ago when he started having constant burning pain and paresthesias in his feet. When he sleeps, the pain often awakens him, and he prefers to uncover his feet because the sheet makes his shins uncomfortable. He has had diabetes mellitus for eight years. He drinks a glass of wine occasionally with dinner, and there is no history of morning drinking, blackouts, binges, tremor, guilt about drinking, or trouble with the law due to drinking. His behavioral brain examination reveals no abnormalities. Vibration and pain sensation are reduced in both feet and ankles, and ankle jerks are decreased bilaterally. 9. Of the following the most likely cause of his symptoms and signs is A) Diabetes mellitus B) Vitamin B12 deficiency C) Thyrotoxicosis D) Multiple sclerosis E) Thiamine deficiency

(A) Diabetes mellitus is the obvious cause, as discussed in item 8. There is no evidence of Vitamin B12 deficiency; for example, there is no evidence of posterior and lateral spinal cord injury. Thyrotoxicosis occasionally causes peripheral neuropathy but here is no evidence of thyrotoxicosis such as exophthalamos, tremor, heat intolerance, elevated diastolic blood pressure, enlarged thyroid, tachycardia or anxiety. Multiple sclerosis affects the central nervous system with patches of demyelination, with exacerbations and remissions at multiple locations, and is not associated with diabetes. Thiamine deficiency can cause peripheral neuropathy, but there is no obvious cause of thiamine deficiency in this case.

A 29-year-old married woman was neuropsychiatrically well until six months ago when, one week after delivering her second child, she started experiencing weakness in opening, closing and moving her eyes, chewing, smiling, pursing her lips, and flexing and extending her neck. Often she has double vision. Her weakness and double vision intensify after she uses the affected muscles multiple times, and intensify later in the day. Her strength improves after she rests. She has bilateral ptosis, and impaired eye abduction or adduction. Her pupils respond normally to light and accommodation. Her language skills are normal, but her voice weakens after sustained conversation. Her gait and deep tendon reflexes are normal, and there is no atrophy or fasciculation. Her Mini-mental State score is 30/30, and her problem-solving, planning and interpersonal abilities are excellent. Patients with this disorder A) Often have disease of the thymus B) Usually have cerebellar dysfunction C) Tend to have corpus striatum disorders D) Have normal neuromuscular junction functioning E) Have normal immune system functioning

(A) Often have disease of the thymus. Thymic tumors occur in 10-15% of myasthenics, and lymphofollicular hyperplasia of the thymic medulla occurs in 65%. Myasthenia gravis often coexists with other autoimmune diseases, including thyrotoxicosis, lupus erythematosus, rheumatoid arthritis, Sjogren syndrome, mixed connective tissue disease, anticardiolipin antibody and polymyositis. Immune function is not normal. The brain is not affected, so there is normal structure and function of the cerebellum and corpus striatum.

A 58-year-old right handed college professor who had a severe closed head injury (Glasgow Coma Scale <10) following a fall two years ago is brought to see you by his wife. His wife is angry because he acts inappropriately in social situations. He often makes crude remarks, shows undue familiarity with strangers, and asks inappropriate personal questions. Her repeated attempts to explain this to the patient have been unsuccessful, and—because he is easily irritated—they often argue about it. Prior to the injury, his self control, interpersonal skill and manners were excellent, and they rarely fought. 9. Of the following, which part of the brain was most likely injured during his fall? A) Orbitomedial prefrontal cortex B) Right parietal cortex C) Right temporo-occipital cortex D) Left parietal cortex E) Left temporo-occipital cortex

(A) Orbitomedial prefrontal cortex, based on his personality and behavior changes, including social inappropriateness and loss of social graces.

A 38-year-old man is referred to a psychiatrist by his family doctor, who thinks the man is "neurotic." The patient complains of being irritable and having headaches, difficulty concentrating, fatigue and sensitivity to sounds. He also sleeps poorly. He denies depression, suicidality or features of psychosis. He has no prior psychiatric history. His general medical health is within normal limits. He was seen in the emergency room six months ago following a car accident in which he was "knocked out" for several minutes and felt dizzy. His neurologic examination and CT scan done at that time were normal. Of the following, which is the most likely diagnosis? A) Post-concussion syndrome B) Epidural hematoma C) Subdural hematoma D) Agoraphobia E) Normal pressure hydrocephalus

(A) Post-concussion syndrome, characterized by a past history of unconsciousness lasting less than 30 minutes, irritability, headaches, difficulty concentrating, fatigue and sensitivity to sounds. Other signs of post-concussion syndrome, not present here, include photophobia, dizziness, and mild non-specific EEG slowing. Acute or subacute subdural or epidural hematoma often present with delirium or coma, either immediately or after a lucid interval, and sometimes presents with pupillary dilatation or hemiparesis. (Lucid intervals are more common in epidural than subdural hematomas, but can occur in both.) If the frontal lobe is injured, frontal signs can appear, including dysexecutive syndrome or orbitomedial disinhibited syndrome.

A 50 year old professor was neuropsychiatrically well until one year ago when she developed a head and hand tremor which make it difficult to eat and drink normally. It is most prominent when she is anxious or works hard. A glass of wine, diaphragmatic breathing, and other relaxing behaviors reduce the tremor. The head tremor makes her look as if she is shaking her head "no." The hand tremor is present only when she uses anti-gravity muscles, not at rest. Besides the tremor, there are no other history or physical findings indicating neuropsychiatric or general medical abnormality. Effective treatments for this condition include A) Propranolol, primidone and gabapentin B) L-DOPA and carbidopa C) Fluoxetine, sertraline and fluvoxamine D) Haloperidol, pimozide and ziprasodone E) Buspirone, diazepam and enalapril

(A) Propranolol or other beta blockers like metoprolol or nadolol and the anticonvulsants primidone and gabapentin are all effective. Amantidine (a dopaminergic antiparkinson antiviral agent) can be a helpful adjunct. Botulinum toxin injections may reduce tremor severity, but the accompanying weakness of arm and hand muscles often is unacceptable to the patient. Although alcohol can reduce the tremor's intensity, when the alcohol wears off, the tremor intensifies. L-DOPA and carbidopa are treatments for Parkinson's disease, not essential tremor. Fluoxetine, sertraline and fluvoxamine are selective serotonin reuptake inhibitors which are valuable for depressions, anxiety disorders and several other conditions but not essential tremor. Haloperidol, pimozide and ziprasodone are antipsychotic, neuroleptic agents that can exacerbate tremor and would not be used for essential tremor. Buspirone is used for generalized anxiety disorder and some behavioral abnormalities associated with certain dementias, but not essential tremor. Diazepam is benzodiazepine which might temporarily alleviate essential tremor but is not an appropriate treatment for it. Clonazepam, another benzodiazepine anxiolytic agent, and valproic acid, an anticonvulsant mood stabilizing agent, may help for orthostatic tremor variant (lower limb) variant of essential tremor, but not the upper limb/head variety.

A 56-year-old right-handed attorney was neuropsychiatrically well until her husband noticed her left arm suddenly became weak and she started bumping into objects. She insists she is perfectly well and can work at the office the next day, and refuses to go to the emergency room. She also asserts her husband is an impostor who is making up stories. She has left ankle clonus, a left Babinski sign, and left arm upper pronator drift. She speaks fluently and comprehends language normally. Of the following, which are her most likely lesion locations'? A) Right frontal and parietal lobes B) Posterior columns, pons and left cerebellar hemisphere C) Left prefrontal and pre-motor cortices D) Left occipital region and corpus callosum E) Right occipital region and corpus callosum

(A) Right frontal and parietal lobes, based on the findings of left arm weakness, left ankle clonus and a left Babinski sign (right frontal lesion), left spatial neglect, anosognosia for her dysfunctions, and Capgras syndrome (right parietal lesion). Posterior column disease causes impaired position and vibration sense and Romberg's sign. Pontine disease can cause a medial inferior pontine syndrome with contralateral hemiparesis; contralateral loss of touch sensation; and lateral rectus paralysis. It can cause a lateral inferior pontine syndrome with ipsilateral facial nerve paralysis, loss of taste from the anterior two thirds of the tongue, ipsilateral loss of lacrimation and reduced salivation, and loss of corneal reflex, unilateral central deafness, nystagmus, nausea, vomiting and vertigo, ipsilateral loss of pain and temperature sensation from the face and contralateral loss of pain and temperature from the trunk and extremities. It can cause medial longitudinal fasciculus syndrome with medial rectus palsy and nystagmus in the abducting eye. It can also cause facial colliculus syndrome with ipsilateral facial paralysis, ipsilateral loss of the corneal reflex, lateral rectus paralysis, and diplopia. Left prefrontal and pre-motor cortex disease often causes avolition; reduced spontaneity; impaired ability to categorize; impaired planning and judgment; problems with set-shifting; disinhibition, utilization behavior, stimulus bound hyperactivity; irritability; and loss of social graces; Broca's aphasia; and impaired motor regulation. Left occipital lesions can cause right homonymous hemianopia; right achromatopsia, and unilateral Balint syndrome. When combined with lesions of the splenium of the corpus callosum, left occipital lesions cause alexia without agraphia. Right occipital lesions can cause left homonymous hemianopia, right achromatopsia and unilateral Balint syndrome.

Asked to "Put your coat back on," a 66-year-old right-handed man who had a stroke puts it on backwards. Asked to copy the outline of a Greek cross with a pen in his right hand, he does so incorrectly. Asked to copy the same outline with his left hand. he also does it incorrectly. These findings indicate abnormality of the A) Right parietal lobe B) Corpus callosum C) Left temporoparietal region D) Left prefrontal cortex E) Right prefrontal cortex

(A) The right parietal lobe contributes to dressing and constructional praxis, left sided kinesthetic praxis, left hand stereognosis and graphesthesis, left spatial recognition, recognition of one's own illness, recognition of others' prosody, and left sided pain, temperature, touch, position and vibration sense. The corpus callosum permits each hemisphere to "know" what the other hemisphere is doing. The left temporoparietal region handles word selection and comprehension along with limbic system memories for correct words; writing, calculating, left-right orientation, finger gnosis; pain, temperature, touch, vibration and position sense from the right side of the body. The left (dorsolateral) prefrontal cortex handles volition, spontaneity, set-shifting, abstraction, categorization, planning and reasoning. The left (orbitomedial) prefrontal cortex contributes to self-control, control of aggression, stimulus resistance and self control, and social graces. The right (dorsolateral) prefrontal cortex contributes to emotional expressivity, expressive prosody and gesturing, and the ability to draw creative shapes fluently. The right (orbitomedial) prefrontal cortex contributes to self-control, control of aggression, stimulus resistance, and social graces.

A 67-year-old hypertensive man reports that yesterday, he had a 5-10 minute episode of severe right eye blurring. One month ago, he had a brief episode of right eye blindness. During both of these episodes his alertness and consciousness were normal. There is no prior history of neuropsychiatric illness. His examination is now normal. 2. Of the following. the most likely diagnosis is A) Transient ischemic attack B) Subarachnoid hemorrhage C) Multiple sclerosis D) Conversion disorder E) Partial complex seizures

(A) Transient ischemic attack (TIA), characterized in this case by brief episodes of right eye blurring or blindness with otherwise normal neuropsychiatric function. TIAs, which are brief, reversible episodes of focal, non-convulsive ischemic neurologic disturbance lasting less than 24 hours, are warning signs of stroke (and also myocardial infarction). Transient monocular blindness (amaurosis fugax), as occurred here, is a typical finding in carotid TIAs. Sometimes the experience is like a shade falling or rising smoothly over the visual field until the eye is completely but painlessly blind. Subarachnoid hemorrhage due to ruptured berry aneurysm is accompanied by sudden severe headache, nausea and nuchal rigidity. Monocular blindness does not result from subarachnoid hemorrhage. Although multiple sclerosis often causes monocular blindness, this blindness usually lasts more than a day and is often accompanied by additional neurologic abnormalities reflecting demyelination in other parts of the central nervous system. In addition to the clinical picture, laboratory findings, including cerebrospinal fluid, MRI, CT and angiography distinguish these two causes of monocular blindness. Conversion disorder, which is not a well-validated diagnosis, is characterized by "pseudoneurologic" symptoms and signs which do not fit any recognized diagnostic pattern, and for which laboratory findings are normal. It is best to avoid this diagnosis, except on USMLE examinations. In partial complex seizures, the patient's alertness and consciousness are impaired, and monocular blindness does not occur.

A 47-year-old married mother of two was neuropsychiatrically well until two weeks prior to clinic examination, when she developed right eye pain. At first, the pain was moderate and occurred intermittently, and then it became intense and continuous. It didn't throb or pulsate. She had no trouble seeing, photosensitivity, aura, nausea, vomiting, scotomata, rash, or tearing. When she looked in the mirror, neither eye was red. A week before the clinic visit, she had three episodes, two days apart, in which "I couldn't get my bearings." The first two involved "the stacks of materials on my desk. I knew what the materials were, but couldn't figure out their relationship to each other. This was weird because I'm well-organized." A similar episode occurred at a department party when, while dancing, "I couldn't decide whether to move my left or right foot." After the episodes were over, her spatial orientation returned to normal. She smokes a pack a day. She had left leg phlebitis after her second childbirth. She took birth control pills when she was in her 20s, but currently takes no medications. She has no history of seizures or diabetes, heart disease or head trauma. She has an identical twin sister with no history of facial pain, transient spatial disorientation, vascular disease, diabetes or seizures. A thorough physical, including behavioral brain and general neurologic examination revealed no neurologic, general medical or psychiatric abnormalities. The key pathophysiologic fact underlying this correct parsimonious conclusion is A) Trigeminal division V1 supplies brain blood vessels and meninges. B) The brain parenchyma responds to intracranial masses with intense pain sensations. C) Visuo-spatial and other cognitive dysfunctions typify cluster headache. D) In addition to migraine sometimes causing cognitive and mood abnormalities, its most common manifestation is eye pain. E) Occipital lobe disease is a frequent cause of eye pain.

(A) Trigeminal division V1 supplies brain blood vessels and meninges, which explains this patient's eye pain. Except for some thalamic syndromes, brain parenchyma, including occipital lobe parenchyma, is not innervated for pain sensation. Cluster headache produces severe pain, usually localized in and around the eye, often with associated with blocked nostrils, rhinorrhea, injected conjunctiva, lacrimation, miosis and flushing or edema in the back of the cheek, all lasting 15-180 minutes. It does not cause visual-spatial and other cognitive abnormalities. Eye pain is rarely if ever a prominent migraine symptom.

A 74-year-old married grandmother was neuropsychiatrically well until three weeks ago when she had a 15-minute episode of left-sided weakness and slurred speech. Two weeks ago, she had sudden impairment of vision of her right eye lasting 10 minutes. Before that, there is no history of neuropsychiatric disorder. Now, her physical examination is essentially normal. Her transient visual loss was most likely due to which of the following? A) Retinal vein thrombosis B) Central retinal artery ischemia C) Posterior cerebral artery ischemia D) Posterior ciliary artery ischemia E) An unconscious conflict

(B) Central retinal artery ischemia, characterized by transient monocular blindness. The central retinal artery arises from the internal carotid artery. Central retinal vein occlusion occurs most frequently with diabetes mellitus, hypertension, leukemia, sickle cell disease, multiple myeloma, macroglobulinemia or an orbital mass, none of which the patient has. It is accompanied by enlarged, tortuous retinal veins and streaky hemorrhages and exudates, none of which this patient has. The posterior cerebral artery usually forms from bifurcation of the vertebral artery and can produce a greater variety of clinical effects than occlusion of any other artery. These include thalamic syndromes, hemiplegia, Weber syndrome (third nerve palsy plus contralateral hemiplegia), eye movement paralysis, pupillary muscle dysfunction, ataxia, tremor, homonymous hemianopia, cortical blindness, achromatopsia, Balint syndrome, dyslexia without agraphia, prosopagnosia, simultanagnosia, visual hallucinations. The posterior ciliary arteries, which arise from the ophthalmic artery distal to the origin of the central retinal artery, supply the choroid and iris. With good evidence for recent vascular occlusion, looking for unconscious conflicts would be unimportant and negligent. Plus, it is impossible to observe an unconscious conflict.

A 68-year old married woman has had weakness—particularly in the eyes, face and neck—double vision and blurry vision over the past five months. The weakness and visual problem are most intense at the end of the day. When she is asked to perform a movement like shrugging her shoulders multiple times in succession, strength declines in those muscles. Her pupils are equal and reactive to light, and her optic discs look normal. Of the following, the best test for the condition is to administer A) Dexamethasone B) Tensilon C) Atropine D) Radiolabelled glucose E) Thymus gland extract

(B) In patients with myasthenia gravis, administering edrophenonium (Tensilon), an acetylcholinerase inhibitor, increases acetylcholine at the neuromuscular junction and temporarily alleviates symptoms. Another test is to screen for serum acetylcholine receptor antibodies. The dexamethasone suppression test is for melancholia—In most melancholics, administering dexamethasone, a synthetic steroid, inhibits ACTH and cortisol production minimally, and serum cortisol is 5.0 or greater at 4 pm after dexamethasone administration. Administering atropine, an anticholinergic agent, to myasthenic patients would increase symptoms by decreasing acetylcholine at the neuromuscular junction. Radiolabelled glucose is administered in PET scans to detect levels of glucose utilization in various brain regions. This would be unhelpful in myasthenia gravis, a neuromuscular junction disease that does not affect the brain.

A 60-year-old right handed man was neuropsychiatrically well until three months ago, when he suddenly developed a severe language problem and right-sided weakness. His alertness and attention are normal. He does not talk unless he is spoken to. He answers questions with a word or phrase but not a complete sentence, and has trouble understanding some of what is said to him. Asked to repeat single words spoken by the examiner, he does so correctly some of the time, and incorrectly some of the time. Asked to repeat "No ifs ands or buts," he replies "No if." He has difficulty distinguishing his right from his left side. He has right arm paralysis, right leg weakness, and right homonymous hemianopia. 11. His lack of spontaneous speech suggests a lesion in his A) Right frontal lobe B) Left frontal lobe C) Right temporoparietal region D) Left occipital lobe E) Corpus callosum

(B) Left frontal lobe disease explains both reduced spontaneity (left dorsolateral prefrontal cortex) and right sided weakness (left motor cortex). Right frontal disease is associated with reduced emotional expressivity, reduced ability to draw designs creatively and fluently (reduced figural fluency), and left sided weakness (right motor cortex disease). Right temporoparietal disease is associated with left sided sensory impairment; impaired prosodic comprehension; constructional and dressing dyspraxia; left hand kinesthetic dyspraxia; left hand stereoagnosia and left hand graphanesthesia. Left occipital disease is associated with right homonymous hemianopia, right hemiachromatopsia, and right Balint syndrome. Corpus callosal disease is associated with inability to tie one's shoelaces with eyes closed; right hand constructional dyspraxia along with normal left hand constructional praxis; left hand ideokinetic dyspraxia along with normal right hand ideokinetic praxis; left hand graphanesthesia along with normal right hand graphesthesis; left hand stereoagnosia along with normal right hand stereognosis; and inability, with closed eyes, to mimic a position, with one hand and wrist, in which the examiner has placed the other hand and wrist.

A 56-year-old right-handed attorney was neuropsychiatrically well until her husband noticed her left arm suddenly became weak and she started bumping into objects. She insists she is perfectly well and can work at the office the next day, and refuses to go to the emergency room. She also asserts her husband is an impostor who is making up stories. She has left ankle clonus, a left Babinski sign, and left arm upper pronator drift. She speaks fluently and comprehends language normally. Of the following, which are her most likely lesion locations'? A) Right frontal and parietal lobes B) Posterior columns, pons and left cerebellar hemisphere C) Left prefrontal and pre-motor cortices D) Left occipital region and corpus callosum E) Right occipital region and corpus callosum

(B) Left spatial neglect and constructional apraxia, left hand stereoagnosia, left hand graphanesthesia, and left hand kinesthetic dyspraxia occur with right parietal lesions. Romberg sign occurs with dorsal column disease, and bilateral intention tremor occurs with bilateral cerebellar disease. Alexia, agraphia, finger agnosia and left-right disorientation—the combination is termed Gerstmann's syndrome—occur with angular gyrus disease. Inability to tie one's shoelaces with closed eyes is associated with corpus callosal disease. Rigidity, mask facies, shuffling and festinating gait are associated with Parkinson's disease.

A 32-year-old computer technician slowly and progressively develops sensations of pins and needles and pain from the wrist to the palm, thumb, forefinger and middle finger bilaterally. Flexion, extension and percussion of the wrist increase the pain. The pins-and-needles sensation is most intense at night. There is mild atrophy of the thenar muscles. Of the following, the most likely diagnosis is A) Axillary nerve neuropathy B) Median nerve neuropathy C) Muscular dystrophy D) Radial nerve neuropathy E) Ulnar nerve neuropathy

(B) Median nerve neuropathy (carpal tunnel syndrome), characterized in this case by pain, numbness and paresthesias from the wrist to the palm, intensified by wrist flexion and thenar atrophy. Radial nerve neuropathy is characterized by foot drop with weakness of ankle aversion and dorsiflexion, and sensory loss along the anterolateral aspect of the leg and dorsum of the foot. This is usually due to damage to the radial nerve as it winds around the humerus. Axillary nerve neuropathy may result from shoulder dislocation, fracture of the neck of the humerus, brachial neuritis and as a response to vaccine or serum. There is wasting of the deltoid muscle and impaired arm abduction. The axillary nerve supplies the deltoid and teres minor muscles.

One hour ago, a 40-year-old hypertensive woman suddenly experienced a headache that she described as the worst headache of her life, trouble concentrating, nausea, light sensitivity and dizziness, followed by a generalized tonic-clinic seizure. She is afebrile, restless and inattentive. Her neck is stiff. She makes 11 errors on her "A" test, and her Mini-mental State score is 14/30. Cerebrospinal fluid is xanthochromic. Of the following, the most likely cause of her condition is A) Bacterial meningitis B) Ruptured aneurysm C) Viral meningitis D) Subdural hematoma E) Migraine headache

(B) Ruptured aneurysm, characterized in this case by sudden profound headache, nausea and light sensitivity, delirium, stiff neck and xanthochromic cerebrospinal fluid. Bacterial and viral meningitis also cause stiff neck, delirium and seizures, but fever is usually present, headache isn't usually profound, and cerebrospinal fluid is clear or cloudy (due to white cells), not xanthochromic. Subdural hematoma can cause delirium, seizures, headache and xanthochromic spinal fluid, but stiff neck is not typical and headache is not usually profound. Migraine headache can cause severe or profound headache (usually unilateral and throbbing), nausea and light sensitivity, but not delirium, seizures, stiff neck or xanthochromic spinal fluid.

A 66-year-old right-handed woman with atrial fibrillation is brought to the emergency room with right sided weakness and labored, slurred speech. Onset occurred suddenly 30 minutes ago while she was washing her face. Previously, she was neuropsychiatrically well. Examination reveals an irregularly irregular heartbeat, right upper extremity and right facial weakness, and slow, labored. dysarthric, telegraphic speech. Her blood pressure is normal, her skin has no petechiae, ecchymoses or hematomas, and her neck is supple. Of the following, the most likely diagnosis is A) Subarachnoid hemorrhage from a ruptured berry aneurysm B) Ischemic stroke due to embolism from the heart C) Arteriosclerotic occlusion of the left internal carotid artery D) Ruptured arterio-venous malformation in the right frontal lobe E) Multiple sclerosis, most severe in the left frontal lobe

(B) Sudden onset of neurologic abnormality in a person with atrial fibrillation strongly suggests cerebral embolism arising from the heart. Subarachnoid hemorrhage from a ruptured berry aneurysm is less likely because of the presence of atrial fibrillation and absence of profound headache or stiff neck. Arteriosclerotic occlusion of the left internal carotid artery can cause an embolus with stroke in any tributary vessel of the internal carotid artery, or may lead to ischemia in the distal field (watershed or border zone) in the region of the lowest perfusion between major branch vessels. Failure of distal perfusion may involve all or part of the middle cerebral artery. The variety of neurologic abnormalities that result from the above is huge, including contralateral paralysis or sensory impairment; Broca's, Wernicke's or global aphasia; Gerstmann's syndrome, right homonymous hemianopia, and paralysis of gaze to the right. That is possible in this case, but with atrial fibrillation, internal carotid occlusion is less likely than embolus from the heart. Ruptured right frontal arteriovenous malformation can cause reduced emotional expressivity, expressive dysprosodia, left hemiparesis, impaired lateral gaze, and signs of increased intracranial pressure. Multiple sclerosis can cause hemiparesis and aphasia, but onset age is typically much younger and there is often a multi-year history of exacerbations and remissions of abnormalities in several parts of the brain or spinal cord.

A 37-year-old woman types on her computer for 5-7 hours a day. She is particularly susceptible to injury of the A) Axillary nerve B) Median nerve C) Ulnar nerve D) Radial nerve E) Long thoracic nerve

(B) The median nerve is often entrapped as it passes together with the tendons of the hand under the flexor retinaculum on the flexor surface of the wrist. It is most common in women over 30 and can be associated with activities like typing or house-painting that cause repetitive stress injury, edema and inflammation in the wrist. Other causes include pregnancy, oral contraceptives, hypothyroidism, arthritis, wrist fracture, acromegaly, uremia, diabetes and amyloidosis. Carpal tunnel syndrome, the entrapment syndrome which follows, is best recognized by weakness of the abductor pollicis brevis, which abducts the thumb perpendicular to the plane of the palm, although thumb flexion and opposition may also be weak. Symptoms often include sensory loss in the first, second and third digits, and prominent paresthesias. Patients often report shaking the hand to try and relieve symptoms (flick sign). In advanced cases, thenar atrophy may occur. Axillary neuropathy is caused by dislocation or fracture of the proximal humerus, causing deltoid weakness and shoulder numbness. Ulnar neuropathy is associated with entrapment of the ulnar nerve at the elbow, with weakness of wrist flexion and adduction, and flexion of the fourth and fifth digits, along with sensory loss and paresthesias in an ulnar distribution. These fingers may assume an "ulnar claw" posture. Radial neuropathy is often caused by sleeping with the arm slung over a park bench, compression in the axilla by improper crutch use, or fracture of the humerus. There is weakness of the extensors of the arm, hand and fingers and loss of the triceps reflexes, and a wrist drop is often present. The long thoracic nerve arises from the fifth, sixth and seventh cervical nerves to the serratus anterior muscle, which fixates the scapula to the chest wall. Paralysis of this muscle results in an inability to raise the arm over the head and winging of the medial border of the scapula when the outstretched arm is pushed forward against resistance. The nerve is most commonly injured by carrying heavy weight on the shoulder or by strapping the shoulder on the operating table. Sometimes this neuropathy follows immunization.

A 67-year-old hypertensive man reports that yesterday, he had a 5-10 minute episode of severe right eye blurring. One month ago, he had a brief episode of right eye blindness. During both of these episodes his alertness and consciousness were normal. There is no prior history of neuropsychiatric illness. His examination is now normal. A) Left middle cerebral artery B) Right posterior cerebral artery C) Right internal carotid artery D) Left posterior inferior cerebellar artery E) Left posterior communicating artery

(C) Branches of the right internal carotid artery supply the right optic nerve and retina, and occlusion typically causes monocular blindness. The left middle cerebral artery, through its cortical branches, supplies the lateral part of the left cerebral hemisphere. Its territory encompasses (1) cortex and white matter of the lateral and inferior parts of the left frontal lobe—including motor areas 4 and 6, contraversive centers for lateral gaze—and Broca's area; (2) cortex and white matter of the left parietal lobe, including the sensory cortex and the angular and supramarginal convolution; and (3) superior parts of the left temporal lobe and insula, including Wernicke's area. The penetrating branches of the middle cerebral artery supply the left putamen, part of the head and body of the caudate nucleus, the outer globus pallidus, the posterior limb of the internal capsule, and the corona radiata. The classic picture of total occlusion of the stem is right hemiplegia, hemianesthesia and homonymous hemianopia (due to infarction of the lateral geniculate body), with deviation of the head and eyes toward the side of the lesion; in addition there is a global aphasia. An embolus entering the middle cerebral artery most often lodges in one of its two main divisions, the superior division (supplying the rolandic and pre-rolandic areas) or the inferior division (supplying the lateral temporal and inferior parietal lobes). Major infarction in the superior division causes a dense sensorimotor deficit in the contralateral face, arm and, to a lesser extent, leg as well as ipsilateral deviation of the head and eyes (i.e., it mimics the syndrome of stem occlusion except that the foot is spared and the leg is less involved than the arm and face). Occlusion of the inferior divison of the middle cerebral artery is less frequent than occlusion of the superior one, and is nearly always due to cardiogenic embolism. The usual result is Wernicke's aphasia. There is also usually a superior quadrantanopia or homonymous hemianopia. In about 70% of persons, both posterior cerebral arteries are formed by the bifurcation of the basilar aertery. In 20-25%, one posterior cerebral artery arises from the basilar in the usual way, but the other arises from the internal carotid. The interpeduncular branches, which arise just above the basilar bifurcation, supply the red nuclei, the substantia nigra bilaterally, medical parts of the cerebral peduncles, oculomotor and trochlear nuclei and nerves, reticular substance of the upper brainstem, decussation of the superior cerebellar peduncles, medical longitudinal fasciculi, and medial lemnisci. The thalamoperforate branches (paramedian thalamic arteries) supply the inferior, medial and anterior parts of the thalamus. The terminal or cortical branches supply the inferomedial part of the temporal lobe and the medial occipital lobe, including visual areas 17, 18 and 19. Occlusion of the posterioor cerebral artery can produce a greater variety of clinical effects than occlusion of any other artery. This could include a thalamic syndrome (e.g., sensory loss in all modalities, spontaneous pain, choreoathetosis), thalamoperforate syndrome (cerebellar ataxia, ipsilateral third nerve palsy), Weber syndrome, contralateral hemiplegia, contralateral ataxic or postural tremor, decerebrate attacks, homonymous hemianopia, cortical blindness, denial of blindness, achromatopsia, Balint syndrome, dyslexia without agraphia, prosopagnosia, simultanagnosia, visual hallucinations and photophobia. The posterior inferior cerebellar artery (Wallenberg) syndrome is a complex of symptoms caused by occlusion of the posterior inferior cerebellar artery or one of its branches supplying the lower portion of the brain stem, resulting in sensory and sympathetic disturbances, cerebellar and pyramidal tract signs, and evidence of partial involvement of the fifth, ninth, tenth, and eleventh cranial nerves. Onset is usually acute with severe vertigo due to involvement of the region of Deiter's nucleus, which may result in falling. Nausea, vomiting, ipsilateral ataxia, muscular hypertonicity, past-pointing, lateropulsion, and other cerebellar signs are often present. Pain and facial paresthesia, a coarse spontaneous homolateral nystagmus, hiccough, dysphagia, dysphonia, dysarthria, and diplopia may be evident. Horner's syndrome is usually present. Sensory disturbances include ipsilateral loss of pain and temperature perception of the face and contralateral hypoesthaesia for pain and temperature of the trunk and extremities. The affected persons have difficulty in swallowing. If there is good collateral circulation, occlusion of one posterior communicating artery may be asymptomatic.

One hour ago, a 40-year-old hypertensive woman suddenly experienced a headache that she described as the worst headache of her life, trouble concentrating, nausea, light sensitivity and dizziness, followed by a generalized tonic-clinic seizure. She is afebrile, restless and inattentive. Her neck is stiff. She makes 11 errors on her "A" test, and her Mini-mental State score is 14/30. Cerebrospinal fluid is xanthochromic. She demonstrates which of the following? A) Dementia B) Extrasylvian sensory aphasia C) Delirium D) Catatonia E) Post-traumatic stress disorder

(C) Delirium, characterized by 11 "A" test errors and other cognitive problems. Although her Mini-mental State score of 14/30 is in the dementia/delirium range, one cannot diagnose dementia in the presence of impaired alertness and delirium, unless the dementia preceded the delirium. Extrasylvian sensory aphasia, which is not associated with impaired alertness, is characterized by spontaneous, fluent paraphasic speech, impaired comprehension, and normal repetition and prosody. Catatonia may be associated with delirium, but there are no catatonic findings (e.g., echopraxia, Mitgehen) here. She has no signs of post-traumatic stress disorder, which doesn't cause seizures or xanthochromic (yellow from red blood cell breakdown) spinal fluid.

A 50 year old professor was neuropsychiatrically well until one year ago when she developed a head and hand tremor which make it difficult to eat and drink normally. It is most prominent when she is anxious or works hard. A glass of wine, diaphragmatic breathing, and other relaxing behaviors reduce the tremor. The head tremor makes her look as if she is shaking her head "no." The hand tremor is present only when she uses anti-gravity muscles, not at rest. Besides the tremor, there are no other history or physical findings indicating neuropsychiatric or general medical abnormality. 15. Of the following, the most likely diagnosis is A) Conversion disorder B) Generalized anxiety disorder C) Essential tremor D) Parkinson's disease E) Parkinsonism due to stroke

(C) Essential tremor is a postural/against gravity/action/extended physiologic tremor most prominent in the hands (bilaterally) or head (with a "nodding 'yes' " or "shaking head 'no' " characteristic), intensified with anxiety and reduced by alcohol. It rarely affects the lower extremities. There is a rare variant, termed orthostatic tremor, in which the lower extremities are primarily affected and is most prominent during quiet standing. If a tremor is unilateral and prominent, consider another disorder, like Parkinson's disease. When it is the only neurologic abnormality in several family members, it is termed familial or hereditary tremor with autosomal dominant inheritance and virtually complete penetrance. If its inherited nature is not evident, is termed essential tremor and, if it becomes evident only in late life, as senile tremor. Sometimes it is mistakenly called benign essential tremor but it is not really benign since it worsens with age and interferes with normal activities. It may appear during teenage or after age 35, with both genders affected. Conversion disorder should not be diagnosed in someone with a classic neurologic disorder like essential tremor, and because of its dubious validity probably shouldn't be diagnosed at all. Mild tremor may occur in generalized anxiety disorder but the latter is associated with chronic anxiety and worry and less prominent than in essential tremor. As you know, the tremor in Parkinson's disease is pill-rolling (with thumb rhythmically apposing forefinger), most prominent at rest, usually unequal bilaterally, sometimes unilateral only, and associated with other findings like rigidity and expressionless face. Parkinsonism due to stroke has many of the same features as Parkinson's disease, but with sudden onset and other signs and CT or MRI evidence of stroke.

Two weeks after having a mild gastrointestinal infection, a 24-year-old woman starts having paresthesias and weakness of her feet. Several days later, she experiences weakness of her legs and thighs. Several days later, she develops weakness of her arms, forearms and hands. The paresthesias and lower extremity weakness persist. Deep tendon reflexes are decreased in all extremities. There is no evidence of a focal transverse spinal cord lesion. Muscle strength does not improve with intravenous infusion of edrophenonium (Tensilon). Cerebrospinal fluid has increased protein but only a few cells. Correct statements about this condition include A) It cannot progress to the trunk, head and brainstem. B) Even if the patient is properly diagnosed and treated, a fatal outcome is inevitable. C) The etiology is probably autoimmune. D) Drug treatment is more important than good nursing and respiratory care. E) Cognitive-behavioral therapy is the treatment of choice.

(C) Etiology of Guillain Barré syndrome is probably autoimmune. It can progress to the trunk, head and brainstem, and respiratory function must be maintained until the patient begins to recover. With proper diagnosis, good nursing care, maintenance of respiratory function, plasma exchange and immune globulin treatment, 95-98% of patients survive and the majority recover (i.e., it is not inevitably fatal). All components of this treatment are crucial. Psychotherapy, including cognitive-behavioral therapy, does not improve the condition. Of course, as in virtually all treatments, a good doctor-patient relationship is important.

A 22-year-old graduate student is brought to the hospital for ascending leg weakness over the past two days. It began with a vague sense of numbness in his feet. Achilles and patellar reflexes are absent, and he has severe weakness in both lower extremities. A) Conversion disorder B) Diabetic polyneuropathy C) Guillain-Barre syndrome D) Myasthenia gravis E) Myotonic dystrophy F) Polymyositis G) Somatization disorder

(C) Guillain-Barré syndrome, characterized initially in this case by paresthesias and numbness of the feet, followed by lower extremity weakness and diminished deep tendon reflexes.

Two weeks after having a mild gastrointestinal infection, a 24-year-old woman starts having paresthesias and weakness of her feet. Several days later, she experiences weakness of her legs and thighs. Several days later, she develops weakness of her arms, forearms and hands. The paresthesias and lower extremity weakness persist. Deep tendon reflexes are decreased in all extremities. There is no evidence of a focal transverse spinal cord lesion. Muscle strength does not improve with intravenous infusion of edrophenonium (Tensilon). Cerebrospinal fluid has increased protein but only a few cells. 12. Of the following, the most likely diagnosis is A) Thyrotoxic neuropathy B) Transverse myelitis C) Guillain-Barre syndrome D) Myasthenia gravis E) Conversion disorder

(C) Guillain-Barré syndrome, characterized initially in this case by paresthesias and weakness of the feet, followed by ascending paralysis affecting the legs and thighs, then the arms, forearms and hands, with diminished deep tendon reflexes. There is no evidence of thyrotoxicosis, such as exophthalamos, tremor, heat intolerance, elevated diastolic blood pressure and increased pulse pressure, enlarged thyroid, tachycardia, atrial fibrillation or anxiety. Myelitis is diagnosed when there is an infectious or non-infective inflammatory process causing destruction of neurons in the spinal cord affecting white matter (tracts), gray matter, or some combination of meninges, white and gray matter. When the gray matter alone is affected, this is termed is poliomyelitis. When white matter alone is affected, the term is leukomyelitis. If a whole cross-section of the cord is affected, this is transverse myelitis. Causes of myelitis include viruses (e.g., poliomyelitis, herpes zoster, HIV); bacterial, fungal parasitic and primary granulomatous diseases (e.g., syphilis, mycoplasma, epidural abscess); and non-infectious causes (e.g., postinfectious myelitis, multiple sclerosis, paraneoplastic myelopathy). Findings include sensory and motor impairments below the level of the lesion. Treatment is based on the cause, and if the cause is an epidural or spinal cord abscess, emergency surgery is usually required. Myasthenia gravis, caused by acetylcholine receptor autoantibodies, typically presents with weakness of extraocular, facial and bulbar muscles, most prominent with repeated use of these muscles, typically at the end of the day. It most often affects young women or older men. Conversion disorder is a "pseudo-neurologic" disorder in which the clinical findings do not fit typical neurologic disease descriptions (e.g., the patient has weakness in the absence of Babinski sign, hyperreflexia, fasciculations, optic atrophy, imaging abnormalities). It is not well validated and should not be diagnosed in the presence of classic neurologic findings like those in this case.

A 67-year-old woman received a diagnosis of lung cancer five months ago. Over the last three months, she has had progressively worsening proximal muscle weakness. There is a purplish discoloration around her eyelids and there are papular erythematous scaly lesions over her knuckles. The purplish discoloration around her eyelids, and the papular erythematous scaly lesions over her knuckles are termed, respectively, A) Lead line and spider angiomata B) Telangectiasias and ecchymoses C) Heliotrope and Gotren's patches D) Shagreen patches and adenoma sebaceum E) Tubors and café au lait spots

(C) Heliotrope (a purplish discoloration about the eyelids) and Gotren's patches (papular erythematous scaly lesions over the knuckles) are characteristic of dermatomyositis. Lead lines are seen in the gums in persons with lead intoxication. Spider angiomata, small red vascular lesions with spidery peripheral extensions, blanch with pressure and are associated with portal hypertension, frequently due to alcoholism. Tubors (in the skin and cerebral cortex) and café au lait spots (flat light brown spots in the skin, occasionally seen in persons without the disease) are associated with Von Recklinghausen's neurofibromatosis, a comparatively common hereditary disease in which the skin, nervous system, bones, endocrine glands and sometimes other organs are the sites of a variety of other congenital abnormalities, often taking the form of benign tumors. There are two forms of the disorder, one associated with a chromosome 17 mutation, and another with a chromosome 22 abnormality. Adenoma sebaceum, tubers and shagreen patches are seen in another neurocutaneous disease, tuberous sclerosis. Adenoma sebaceum are angiofibromas consisting of nodules several millimeters in diameter, firm and pale, on the malar surface of the face in a butterfly distribution that appear in teenage and sometimes resemble acne. Tubers are potato-like brain nodules, 1-3 cm in diameter, that grow to compress and irritate the surrounding cortex, causing epilepsy and cognitive impairment, and occasionally undergo malignant transformation. When tuberous sclerosis occurs in infants and children, it frequently manifests leathery scaly trunk lesions termed shagreen patches. Tuberous sclerosis is associated with abnormalities of chromosomes 9 or 16.

A 72-year-old man with a long history of hypertension was neuropsychiatrically well until four days ago when he began to experience headache slightly worse on the right, bumping into objects, and inability to dress himself. Examination on the fourth hospital day is as follows: Blood pressure is 140/100. The left side of his face is unshaven. He has left sided numbness, left homonymous hemianopia, and dressing dyspraxia. Asked to draw a clock with the hands at 11:15, he places all the numbers on the right side of the drawing. On the day of admission, CT of his brain revealed a white, hyperintense oval lesion in the right parietal lobe. A) Cerebral embolus with infarction B) Epidural hematoma C) Intracerebral hemorrhage D) Ruptured saccular aneurysm E) Subdural hematoma

(C) Intracerebral hemorrhage, based on the hyperintense oval lesion in the right parietal lobe, which fits with the findings of dressing dyspraxia and left spatial neglect.

A 60-year-old right-handed man was neuropsychiatrically well until his wife noticed that suddenly, his speech slurred and one corner of his mouth drooped. His left naso-labial fold is more clearly defined than the right one. He can furrow both sides of his forehead and close his eyes tightly, symmetrically and bilaterally. He has right ankle clonus. Mini-mental State score is 26/30. 8. Of the following, the most likely cause of these findings is A) Bell's palsy B) Trigeminal neuralgia C) Stroke D) Alzheimer's disease E) Cruetzfeld-Jacob disease

(C) Left hemisphere stroke, characterized by lesions of the left corticobulbar (central facial palsy with sparing of the upper and middle face) and left corticospinal tracts (ankle clonus) prior to their crossing the midline. In Bell's palsy, with lesions of the facial nerve, all motor divisions of the seventh (facial) nerve are affected. Trigeminal neuralgia (tic douloureux) is a chronic recurring condition in which patients suffer from dozens of brief 20-30-second jabs of excruciatingly sharp pain that extend along one of the three divisions of the trigeminal nerve, most often V2. Unlike all other headaches, stimulating the affected area can provoke the pain. These regions, trigger zones—when stimulated by eating, touch, brushing teeth, or drinking coldwater—evoke a dreadful shock. During a severe attack, patients hesitate to eat, brush their teeth and speak. Fortunately, it abates at night, allowing a full night's sleep. It typically develops after age 60. Often, the cause is an aberrant superior cerebellar artery or other cerebral blood vessel compressing the trigeminal nerve root as it emerges from the brain stem. Carbamazepine is usually the initial treatment. Alzheimer's disease is the most common of all the dementing illnesses, with incidence and prevalence increasing with age. It develops gradually over multiple years, initially affecting temporoparietal regions with impaired memory (that doesn't improve with cuing) and getting lost without initial gait alterations or personality changes. Early findings include temporoparietal hypoperfusion on SPECT and digit span and digit symbol substitution abnormalities. Cruetzfeld-Jacob disease (CJD) is the most common prion disease. Some 85% of cases are sporadic (sCJD), most of the rest familial (fCJD). There is also a new variant (vCJD). Small percentages are due to iatrogenic due to exposure to infected human tissues. It is rare, with average age of onset 61.5 years, range 20-79. It has a short, lethal course, with average survival 4-6 months. Features include dementia, myoclonus, cerebellar ataxia, and pyramidal and extrapyramidal tract signs. It may also include painful neuropathy, choreiform movements, hallucinations, cortical blindness, aphasia, supranuclear gaze palsy, and the alien hand syndrome. Diagnostically, the best CSF marker for sCJD is presence of normal cellular proteins designated 14-3-3, reflecting extensive destruction of brain tissue. On T2-weighted and proton-dense images, sCJD is often associated with high signal changes, usually symmetric, in the putamen and head of the caudate nucleus. There is a pathognomonic MRI change in vCJD, a "pulvinar sign," characterized by hyperintensity of the pulvinar nucleus of the thalamus relative to the caudate, putamen and cortex. There can also be "hockey stick" appearance from combined hyperintensities of the pulvinar plus the dorsomedial nuclei of the thalamus. Only standard universal precautions are indicated in the management, because the risk of transmission is low. Physicians should not avoid obtaining biopsies and autopsies for fear of transmission. Caution, however, should be used in obtaining CSF and handling tissues obtained at autopsy. No treatment is successful, but quinacrine, chlorpromazine, and flupirtine may help.

A 68-year old married woman has had weakness—particularly in the eyes, face and neck—double vision and blurry vision over the past five months. The weakness and visual problem are most intense at the end of the day. When she is asked to perform a movement like shrugging her shoulders multiple times in succession, strength declines in those muscles. Her pupils are equal and reactive to light, and her optic discs look normal. 17. Of the following, the most likely diagnosis is A) Multiple sclerosis B) Polymyositis C) Myasthenia gravis D) Peripheral neuropathy E) Cerebrovascular accident

(C) Myasthenia gravis, caused by acetylcholine receptor autoantibodies, typically presents with extraocular, facial and bulbar muscle weakness, most prominent with repeated use of these muscles, typically at day's end. It most often affects young women or older men. Polymyositis, characterized most prominently by proximal muscle weakness with tenderness, is autoimmune and often co-exists with systemic lupus erythematosus. Multiple sclerosis is one of the most important neurologic diseases due to its frequency, chronicity and tendency to attack young adults. The neurologic manifestations are protean, being determined by the varied location and extent of the demyelinative foci. Classic features include motor weakness, paraparesis, paresthesias, impaired vision (with optic neuritis and atrophy), diplopia, nystagmus, dysarthria, intention tremor, ataxia, impairment of deep sensation and bladder dysfunction. A long period of latency (1-10 years or longer) between a minor initial symptom and subsequent development of characteristic symptoms and signs may delay the diagnosis. In most cases the initial manifestations improve partially or completely, to be followed after a variable interval by the recurrence of the same abnormalities or the appearance of new ones in other parts of the nervous system. In as many as half the patients, the disease takes the form of an intermittently progressive and sometimes of a steadily progressive one, especially in patients over 40. Although a definitive etiology is unknown, much is known epidemiologically, including that it is more common in higher latitudes (e.g., Canada, upper US) than equatorial ones, that there is some familial vulnerability and proneness to have certain histocompatibility antigens. MRI (the best laboratory test for MS), CT and autopsy show patches of demyelination. Evoked potentials are abnormally slow.

A 60-year-old right handed man was neuropsychiatrically well until three months ago, when he suddenly developed a severe language problem and right-sided weakness. His alertness and attention are normal. He does not talk unless he is spoken to. He answers questions with a word or phrase but not a complete sentence, and has trouble understanding some of what is said to him. Asked to repeat single words spoken by the examiner, he does so correctly some of the time, and incorrectly some of the time. Asked to repeat "No ifs ands or buts," he replies "No if." He has difficulty distinguishing his right from his left side. He has right arm paralysis, right leg weakness, and right homonymous hemianopia. His right homonymous hemianopia suggests a lesion in his A) Right parietal cortex B) Left retina C) Left optic radiations D) Right hemisphere homologue of Wernicke's area E) Cerebellar vermis and right cerebellar cortex

(C) Right homonymous hemianopia is associated with disease of the left optic radiations or the right occipital cortex. Severe left retina disease causes monocular impaired vision or blindness. Disease of the right temporoparietal homologue of Wernicke's area is associated with impaired comprehension of others' prosody. Cerebellar vermis disease is associated with truncal ataxia. Right cerebellar cortex disease is associated with ipsilateral incoordination.

The doctor asks his 62-year-old right-handed patient who had a stroke to "Repeat after me, Oregon, New York, New Jersey," and the patient does so correctly. Immediately afterwards, the doctor instructs the patient to 'Remember these three states because I will ask you about them a few minutes later." Five minutes later, the doctor says, "What were those three states I asked you to remember?" and the patient replies "New Jersey, Oregon, New York." Then the doctor says, "Which of those three states doesn't belong with the other two?" and the patient says, "Oregon, because it's on the West Coast." These findings suggest normal functioning of the patient's left temporal lobe and hippocampus. and his A) Right parietal lobe B) Right motor cortex C) Left prefrontal cortex D) Corpus callosum E) Left occipital region

(C) The left (dorsolateral) prefrontal cortex handles abstraction and categorization, as is the case here. It also handles spontaneity, volition, set-shifting, planning and reasoning. The right parietal lobe attends to the left side of space; handles constructional and dressing praxis; facial recognition; recognition of others' prosody, left hand kinesthetic praxis; left hand stereognosis and graphesthesis, and left-sided pain, temperature, touch, position and vibration sense. The right motor cortex handles movement on the left side of the body. The corpus callosum permits each hemisphere to "know" what the other hemisphere is doing. The left occipital region handles visual sensations and perceptions from the right side of space.

A 47-year-old married mother of two was neuropsychiatrically well until two weeks prior to clinic examination, when she developed right eye pain. At first, the pain was moderate and occurred intermittently, and then it became intense and continuous. It didn't throb or pulsate. She had no trouble seeing, photosensitivity, aura, nausea, vomiting, scotomata, rash, or tearing. When she looked in the mirror, neither eye was red. A week before the clinic visit, she had three episodes, two days apart, in which "I couldn't get my bearings." The first two involved "the stacks of materials on my desk. I knew what the materials were, but couldn't figure out their relationship to each other. This was weird because I'm well-organized." A similar episode occurred at a department party when, while dancing, "I couldn't decide whether to move my left or right foot." After the episodes were over, her spatial orientation returned to normal. She smokes a pack a day. She had left leg phlebitis after her second childbirth. She took birth control pills when she was in her 20s, but currently takes no medications. She has no history of seizures or diabetes, heart disease or head trauma. She has an identical twin sister with no history of facial pain, transient spatial disorientation, vascular disease, diabetes or seizures. A thorough physical, including behavioral brain and general neurologic examination revealed no neurologic, general medical or psychiatric abnormalities. 24. The clinic doctor should tell her A) "Probably there's nothing seriously wrong. It's probably your nerves. Just to be sure, let's do a panel of laboratory studies." B) "There's nothing seriously wrong. You're under a lot of stress. You should consider taking a few days off. Just to be sure, let's do some lab studies. Do you think your boss will let you off from work?" C) "I'm sorry you're having so many symptoms and so much discomfort. This must be very difficult for you." D) "I'm concerned about your symptoms. Let me explain some more, and then let's discuss your having some lab tests today." E) "Your differential diagnosis is between a seizure disorder, a stroke, and a tumor."

(D) "I'm concerned about your symptoms. Let me explain some more, and then let's discuss your having some lab tests today" is an empathic, medically appropriate and responsively prompt statement to this patient with symptoms suggesting a transient ischemic episode. "There's nothing seriously wrong. You're under a lot of stress. Consider taking a few days off" is a cliché'd, inaccurate stereotypic, unhelpful response that doesn't address the patient's situation. "Probably there's nothing seriously wrong, it's probably your nerves. Let's do some laboratory studies" is also inaccurate, cliché'd and stereotypic. "I'm sorry you're having so many symptoms. This must be very difficult" is empathic but doesn't address what needs to be done. "Your differential diagnosis is between a seizure disorder, a stroke, and a tumor" is an unnecessarily dramatic, imprecise and scattershot response. Note that items 25-27 are from the live patient interview by Dr. Welch last January.

A 47-year-old homeless man reports burning sensations in his feet. His history includes morning drinking, fighting while drinking, shaking if he can't have a drink, and forgetting what he did while drinking. Vibration, position, pain and temperature senses are reduced in his lower extremities from his feet to his knees. Of the following, the most likely etiology is A) Diabetes B) Guillain-Barré syndrome C) Femoral nerve injury D) Alcoholism E) Heavy metal toxicity

(D) Alcoholism, characterized in this case by morning drinking, fighting while drinking, withdrawal tremors and blackouts. Alcoholic neuropathy, associated with thiamine deficiency, is characterized by impaired position and vibration sense (with ataxia when walking in the dark) and absent deep tendon reflexes. Diabetic neuropathy is primarily a sensory neuropathy, with loss of sensation and deep tendon reflexes in the feet and ankles, and often manifests painful foot paresthesias, especially distressing at night. Tricyclic antidepressants, and the anticonvulsant gabapentin, help alleviate these symptoms. Although sometimes idiopathic, Guillain-Barré syndrome typically follows an upper respiratory or gastrointestinal illness. Most common in young and middle-aged adults, the syndrome begins with paresthesias and numbness of the fingers and toes and then flaccid paralysis of the feet and legs. Weakness, which then becomes a much greater problem than numbness, ascends to involve the hands and arms. As the disease progresses, the phrenic and intercostal nerves weaken the patient must be intubated for respirator assistance, and the patient also develops cranial nerve palsies with dysphagia, facial weakness and ocular immobility. Cerebrospinal fluid reveals elevated concentration but few white cells (albuminocytologic dissociation). The syndrome resolves within three weeks to three months. The femoral nerve is formed from the second (to a lesser extent), third and fourth lumbar roots, and enters the thigh lateral to the femoral artery. Its anterior division carries sensation from the anteromedial surface of the thigh, and the posterior division provides the motor innervation to the quadriceps. Following femoral nerve injury, there is weakness of extension of the lower leg, quadriceps wasting, failure of knee fixation, and absent knee jerks. It may also lead to weakness of hip flexion. The most common cause of femoral neuropathy is diabetes mellitus. Heavy metal toxicity, discussed in detail in item 4, produces a combination of dementia and peripheral neuropathy.

Doctor: Tell me what you see in this picture. Patient: There's a woman. Doctor: Anything else you see in the picture? Patient: A woman. Doctor: Anything else? Patient: (Does not respond) Doctor: Anything else in the picture? Look all around the picture. Patient: I see the boy. Doctor: Repeat after me, say "No ifs ands or buts." Patient: No ifs ands or buts. A) Broca's aphasia B) Conduction aphasia C) Extrasylvian (transcortical) sensory aphasia D) Extrasylvian (transcortical) motor aphasia E) Global aphasia F) Pure word deafness G) Wernicke's aphasia

(D) Extrasylvian (transcortical) motor aphasia, characterized by reduced spontaneity, normal repetition, reasonably good comprehension, and no paraphasic speech.

Doctor: Tell me what you see in this picture. Patient: Tell me what is in the picture. Doctor: Repeat after me, say 'no ifs ands or buts.' Patient: Repeat after me, say no ifs ands or buts. Doctor: What is in the picture? Patient: In the picture it's too early to tell how many gave them that. Doctor: Tell me, what's happening in the picture? Patient: In the picture nine or ten houses are too old for them. A) Broca's aphasia B) Catatonia C) Circumstantiality D) Extrasylvian sensory aphasia E) Normal language, speech and cognitive function, no abnormality present F) Spatial neglect G) Wernicke's aphasia

(D) Extrasylvian sensory aphasia, characterized by normal repetition with echolalia, normal spontaneity and fluency, impaired comprehension, and paraphasic speech

. A 60-year-old woman reports episodes of double vision and drooping eyes that worsen as each day progresses. Recently, she has had trouble reaching up to brush her hair, and trouble climbing stairs.A) Conversion disorder B) Diabetic polyneuropathy C) Guillain-Barre syndrome D) Myasthenia gravis E) Myotonic dystrophy F) Polymyositis G) Somatization disorder

(D) Myasthenia gravis, characterized in this case by ocular muscle weakness intensified by using the affected muscles multiple times and later in the day; and normal cognitive and pupillary function and deep tendon reflexes.

A 29-year-old married woman was neuropsychiatrically well until six months ago when, one week after delivering her second child, she started experiencing weakness in opening, closing and moving her eyes, chewing, smiling, pursing her lips, and flexing and extending her neck. Often she has double vision. Her weakness and double vision intensify after she uses the affected muscles multiple times, and intensify later in the day. Her strength improves after she rests. She has bilateral ptosis, and impaired eye abduction or adduction. Her pupils respond normally to light and accommodation. Her language skills are normal, but her voice weakens after sustained conversation. Her gait and deep tendon reflexes are normal, and there is no atrophy or fasciculation. Her Mini-mental State score is 30/30, and her problem-solving, planning and interpersonal abilities are excellent. 10. Of the following, the most likely diagnosis is A) Amyotrophic lateral sclerosis B) Partial complex seizures C) Guillain-Barre syndrome D) Myasthenia gravis E) Duchenne's muscular dystrophy

(D) Myasthenia gravis, characterized in this case by young adult onset in a woman, ocular and bulbar muscle weakness intensified by using the affected muscles multiple times and later in the day; and normal cognitive and pupillary function and deep tendon reflexes. Amyotrophic lateral sclerosis typically begins after age 50, affects upper motor neurons with paralysis and hyperreflexia and lower motor neurons with weakness, atrophy and fasciculations. Partial complex seizures are often preceded by auras (simple partial seizures like hallucinations), then characterized by several-minute-long periods of reduced alertness and consciousness, staring and stereotypic movements, followed by postictal confusion. Guillain-Barré syndrome, an acute imflammatory or post-infectious demyelinating polyneuropathy most common in young or middle-aged adults, typically begins with numbness and paresthesias of the fingers and toes, followed by areflexic, flaccid paralysis of the feet and legs. Weakness, which becomes a much greater problem than numbness, ascends to involve the hands and arms. Many patients develop respiratory distress from involvement of the phrenic and intercostal nerves and must be intubated for respirator assistance. If weakness ascends further, patients develop cranial nerve involvement that leads to dysphagia and other types of bulbar palsy, facial weakness and ocular immobility. Because optic and acoustic nerves are protected by myelin generated by the CNS—not the peripheral nervous system—patient continue to see and hear. Also noteworthy is that bladder, bowel and sexual function are preserved, typical of neuropathies except for diabetic neuropathy. The CSF shows albuminocytologic dissociation, with elevated protein but few white cells. The illness usually resolves completely within 3 weeks to 3 months as the peripheral nervous system myelin is regenerated. Duchenne's muscular dystrophy is characterized in by childhood onset of proximal muscle weakness; using Gower's maneuver (arising by using one's arms to push up against the floor and then "climb one's own body" by grasping and pushing up on one's knees and thighs); enlarged but very weak shoulders and calves (pseudohypertrophy); and a family history suggesting X-linked genetic transmission. Muscle biopsy would show little or no dystrophin, and blood testing would show DNA deletions on the X chromosome.

A 29-year-old right handed woman was neuropsychiatrically well until two months ago when she was in an auto accident in which her head struck the windshield and she was temporarily unconscious. She recovered fully except for the fact that she cannot smell or taste what she eats. The most likely cause of this symptom is A) Conversion disorder B) Somatization disorder C) Temporal lobe epilepsy D) Olfactory rootlet injury E) Left parietal lobe contusion

(D) Olfactory rootlet injury, injuring her first cranial (olfactory) nerve, interfering with her ability to smell and taste. Do not diagnose conversion disorder in the presence of a clear neurologic condition, as in this case. In fact, because of the dubious validity of conversion disorder, best not to diagnose it at all. Somatization disorder is ruled out because there is no history of multiple medically unexplained symptoms in multiple systems. Temporal lobe simple 7 partial seizures include sudden unexplained intense moods, including sadness, elation, fear anxiety or panic; and violence; perceptual disturbances, including hallucinations, dysmegalopsia, dysmorphopsia, déjà vu, jamais vu, depersonalization, derealization; and autonomic-visceral disturbances including body temperature instability, cardiovascular signs, epigastric rush, nausea, abdominal pressure and choking sensations; and hallucinations of foul odors if the uncus is affected. Left parietal contusion in a right hander could cause dyscalculea, left-right disorientation, finger agnosia, dysgraphia (if the angular gyrus is affected these four taken together would form Gerstmann's syndrome), right hand kinesthetic dyspraxia, right hand stereoagnosia, right hand graphanesthesia, ideokinetic dyspraxia with either hand, conduction aphasia, and left hand hemianesthesia.

. A 60-year-old man had sudden onset of right-sided hemi-sensory loss in all modalities, plus spontaneous pain and dysesthesias in these same affected regions. A) Anterior cerebral artery syndrome B) Basilar artery syndrome C) Middle cerebral artery syndrome D) Posterior cerebral artery syndrome E) Posterior inferior cerebellar artery syndrome

(D) Posterior cerebral artery syndrome, based on the "thalamic syndrome" findings of hemi-sensory loss, spontaneous pain and dysesthesias. The posterior cerebral artery supplies the thalamus.

Following a head injury, a 45-year-old patient whose eyes were closed opens his eyes when spoken to, obeys requests to move his extremities, and is oriented to time and place. His Glasgow Coma Scale score is A) 1 B) 4 C) 7 D) 10 E) 14

(E) 14, based on 3 points for eye opening to speech (maximum 4 for eyes spontaneously open), 6 points (maximum score) for obeys requests, and 5 points (maximum) for being oriented to time and place. The Glasgow Coma Scale score is computed as follows: Function Description Points Eye opening (E) Opens eyes spontaneously 4 Opens eyes in response to voice 3 Opens eyes in response to pain 2 Does not open eyes 1 Best motor response (M) Obeys commands 6 Localizes painful stimuli 5 Withdraws/flexes to pain 4 Abnormal flexion to pain 3 Extensor response to pain 2 Makes no movements 1 Verbal response (V) Oriented, converses normally 5 Confused and disoriented 4 Utters inappropriate words 3 Makes incomprehensible sounds 2 Makes no sounds 1

A 44-year-old married mother of two was neuropsychiatrically well until two weeks ago, when her brother was murdered. Since then, she has believed that Martians are trying to kill her. She says that the reason for her belief about Martians is that she hears voices speaking in a language that she cannot understand. She has no past or family history of psychiatric illness. Since age 24, she has been hospitalized multiple times for complications of systemic lupus erythematosus, including miscarriages, heart failure, skin lesions, arthritis and kidney disease. She is alert and cooperative, with no motor or language abnormalities. Of the following, the most likely cause of her delusion is A) Schizoaffective disorder B) Delusional disorder C) Multiple sclerosis D) Post traumatic stress disorder E) An autoimmune disease

(E) An autoimmune disease, specifically systemic lupus erythematosus. Applying the principle of parsimony, the presumed pathophysiology is that the stress of her brother's death exacerbated her immune system disease, causing lupus cerebritis. In the presence of a general medical disease that can affect brain functioning, it is not parsimonious to diagnose schizoaffective disorder or delusional disorder or any other primary psychiatric disorder. She has no signs of post-traumatic stress disorder, which does not cause delusions.

A 74-year-old married grandmother was neuropsychiatrically well until three weeks ago when she had a 15-minute episode of left-sided weakness and slurred speech. Two weeks ago, she had sudden impairment of vision of her right eye lasting 10 minutes. Before that, there is no history of neuropsychiatric disorder. Now, her physical examination is essentially normal. 22. Of the following, which would be the best diagnostic test for her condition? A) Continuous electrocardiographic monitoring B) Lumbar puncture C) Electroencephalogram D) Screening for conversion or somatization disorder E) Carotid Doppler ultrasound

(E) Carotid Doppler ultrasound is indicated because the findings suggest left internal carotid artery disease. With this in mind, in the absence of chest pain, dyspnea, edema, or irregularly irregular heartbeat, a single ECG might still be helpful, but continuous ECG monitoring is not necessary. Lumbar puncture might be informative, but in the absence of signs of meningitis, encephalitis or ruptured berry aneurysm, LP will have a lower "yield" than would carotid Doppler investigation. Because the patient has had transient episodes of abnormal behavior, EEG might be useful, but the clinical picture is not classic for epilepsy, so EEG would have a lower yield than carotid Doppler investigation. There is no evidence of somatization disorder (long-time history of medically unexplained symptoms in multiple systems) and, not only is conversion disorder of dubious validity to begin with, it would be negligent and dangerous to diagnose that in someone with a classic TIA syndrome.

A 20-year-old man has chronically worsening distal weakness of his hands and feet. He also has ptosis and frontal balding. A) Conversion disorder B) Diabetic polyneuropathy C) Guillain-Barre syndrome D) Myasthenia gravis E) Myotonic dystrophy F) Polymyositis G) Somatization disorder

(E) Myotonic dystrophy, characterized in this case by young adult onset, prominent facial and peripheral muscle weakness, and frontotemporal balding, giving the appearance of a "hatchet face" (facial atrophy, sunken and elongated face, ptosis and a prominent forehead). The distal muscle involvement distinguishes it from Duchenne's dystrophy and many other myopathies, where proximal muscle involvement predominates.

Following a head injury, a 34-year-old patient's Glasgow Coma Scale score is 5. The likelihood of his dying as a consequence of the injury is approximately A) Zero B) 4% C) 8% D) 12% E) 40%

(E) Persons with total Glasgow Coma Scale scores 3 (minimum)-8 have a 40% chance of dying from the injury. Prognosis based on Glasgow Coma Scale score is as follows: Glasgow Coma Scale Score Severity Mortality 13-15 Mild 4% 9-12 Moderate 4% 3-8 Severe 40%

A 67-year-old woman received a diagnosis of lung cancer five months ago. Over the last three months, she has had progressively worsening proximal muscle weakness. There is a purplish discoloration around her eyelids and there are papular erythematous scaly lesions over her knuckles. 20. Of the following, the most likely diagnosis is A) Myasthenia gravis B) Trichinosis C) Psoriasis D) Progressive multifocal leukoencephalopathy E) Dermatomyositis

(E) Polymyositis, an autoimmune disease that is sometimes associated with a benign viral illness, sometimes with a collagen disease like lupus erythematosus, and sometimes with a pulmonary or gastrointestinal malignancy, is characterized by proximal muscle weakness and sometimes fever and malaise. When it is combined by a purplish discoloration about the eyelids (heliotrope) and papular erythematous scaly lesions over her knuckles (Gotren's patches), it is termed dermatomyositis. Trichinosis, a type of polymyositis, is a trichinella infection of muscles caused by eating undercooked pork or game. Psoriasis is a chronic dermatitis associated with silvery gray and red lesions primarily over the elbows and knees. Myasthenia gravis, which is caused by acetylcholine receptor autoantibodies, typically presents with weakness of extraocular, facial and bulbar muscles, most prominent with repeated use of these muscles, typically at the end of the day. It most often affects young women or older men. Progressive multifocal leukoencephalopathy, an infection, leads to CNS demyelination. It is a late complication of AIDS or other diseases with immunologic impairment. Like multiple sclerosis, it causes patchy demyelination throughout the CNS.

A 35-year-old man has a one-week history of painful muscles. He has a past history of systemic lupus erythematosus with nephritis. His proximal muscles are tender to palpation. A) Conversion disorder B) Diabetic polyneuropathy C) Guillain-Barre syndrome D) Myasthenia gravis E) Myotonic dystrophy F) Polymyositis G) Somatization disorder

(F) Polymyositis is characterized most prominently by proximal muscle weakness with tenderness, is autoimmune and often co-exists with systemic lupus erythematosus. Conversion disorder is a "pseudo-neurologic" disorder in which the clinical findings do not fit typical neurologic disease descriptions (e.g., weakness in the absence of Babinski sign, hyperreflexia, fasciculations, optic atrophy, imaging abnormalities). It is not well validated and should not be diagnosed in the presence of classic neurologic findings like those in this case. Chronic progressive distal diabetic polyneuropathy is the most common type of diabetic neuropathy. It is a mixed sensory-autonomic-motor polyneuropathy—predominantly sensory. In about 10% of patients, pain is the most prominent feature. The sensory symptoms begin distally in the toes extending to the forelegs before involving the upper limbs. Most patients exhibit evidence of both small and large fiber involvement.

A 24-year-old woman with no prior pregnancies has had a hysterectomy for dysfunctional uterine bleeding and an exploratory laparoscopy for which no abnormality was found. She has had transient blindness, shoulder, back, hip and joint pain that could not be medically explained, and long-standing nausea and lack of interest in sex for which no satisfactory explanation was given. A) Conversion disorder B) Diabetic polyneuropathy C) Guillain-Barre syndrome D) Myasthenia gravis E) Myotonic dystrophy F) Polymyositis G) Somatization disorder

(G) Somatization disorder, characterized in this case by multiple medically-unexplained symptoms in multiple systems, including medically-unexplained pain, genitourinary, pseudoneurologic and gastrointestinal symptoms.

Doctor: Tell me what you see in the picture. Patient: A woman is washing her dishes. Doctor: Anything else? Patient: The sink is overflowing. It's light outside. Doctor: Anything else? Patient: No. Doctor: What about the children? Patient: I don't understand. A) Broca's aphasia B) Conduction aphasia C) Extrasylvian (transcortical) sensory aphasia D) Extrasylvian (transcortical) motor aphasia E) Global aphasia F) Pure word deafness G) Wernicke's aphasia

(H) Left spatial neglect, characterized by the patient's non-recognition of anything on the picture's left side. The patient's spontaneity, fluency, and comprehension are normal and there is no paraphasic speech. (I apologize that there was no choice [H] on this item.)

Doctor: Tell me what you see in the picture. Patient: There's a woman, maybe in her thirties—I guess she could be in her forties, also—who's washing the dishes. Looks like glass or plastic dishes, probably not china, probably not too expensive. The sink is overflowing. It's almost a flood. She's not paying nearly as much attention as she should. Reminds me of television shows from the fifties. Doctor: Anything else? Patient: A boy, probably it's her son, is just about to fall from the stool. He's reaching high in the cabinet for a cookie jar. His sister is aiding and abetting him. My best bet is that the cookies are chocolate chip, but it could be sugar or M and M cookies also. This isn't a typical family. Most families are more attentive. A) Broca's aphasia B) Conduction aphasia C) Extrasylvian (transcortical) sensory aphasia D) Extrasylvian (transcortical) motor aphasia E) Global aphasia F) Pure word deafness G) Wernicke's aphasia

(I) Circumstantiality, characterized by the patient's using excessive details, with normal spontaneity, fluency and comprehension, and no paraphasic speech. (I apologize that there was no choice [I] on this item.)

A 19-year-old man was neuropsychiatrically well, with no prior history of headache, until three hours ago when he suddenly awakened with an "incredibly bad headache." Soon after that he vomited and felt neck stiffness. Several minutes later he fell, complained of dizziness and did not move his left side. On examination his alertness is reduced, Mini-mental State score is 10/30, and he has nuchal rigidity, left hemiparesis and a left Babinski sign. Of the fallowing, the most likely diagnosis is A) Migraine headache B) Cluster headache C) Ruptured aneurysm D) Bacterial meningitis E) Viral meningitis

1. (C) Ruptured berry aneurysm, characterized in this case by sudden onset of profound headache, delirium, nausea, nuchal rigidity, left hemiparesis and a Babinski sign. All these typify ruptured berry aneurysms except for hemiparesis and a Babinski sign, which are much more common 3-12 days after the rupture (due to vasospasm) than immediately after the rupture, although it can occur immediately afterwards (perhaps due to a transitory fall in pressure in the circulation distal to the ruptured aneurysm). Bacterial or viral meningitis can cause headache, delirium, nausea and nuchal rigidity, but they are usually accompanied by fever, and not usually associated with profound headache. Hemiparesis with meningitis is rare. Migraine headache is often severe and accompanied by nausea and vomiting, and can be associated with transient hemiplegia (hemiplegic migraine). However, nuchal rigidity and delirium are not typical of migraine, and migraine headache is usually unilateral. Cluster headache also produces severe pain, usually localized in and around the eye, often with associated with blocked nostrils, rhinorrhea, injected conjunctiva, lacrimation, miosis and flushing or edema in the back of the cheek, all lasting 15-180 minutes. It is not associated with nuchal rigidity, delirium or hemiplegia.

A 58-year-old married writer, with a 30-year history of heavy drinking with blackouts, morning drinking and gastritis, is brought by his wife to the hospital because of trouble standing and walking of four weeks' duration. He is cooperative, his history is credible and consistent, and he does not try to intimidate the staff. Currently he cannot stand or walk. Muscle strength in his feet and legs is severely reduced, calves are atrophic, ankle jerks are absent, position and vibration sense in his toes are impaired, plantar reflexes are flexor, and there is no ankle clonus. 1. Of the following, the most likely cause of his lower extremity findings is A) Malingering B) Factitious disorder C) Conversion disorder D) Neuropathy E) Korsakoff s psychosis

1. (D) Neuropathy, characterized in this case by muscle weakness in the feet and legs, atrophic calves, absent ankle jerks and impaired position and vibration sense. The findings point to peripheral nerve disease. Alcoholism is the obvious cause. There is no evidence, such as from clonus or Babinski sign, of an upper motor neuron lesion. There is no evidence for malingering: There is no legal or financial situation in the background. Although he is a substance abuser and substance abusers malinger more often than the general population, there is no evidence he is lying: The findings fit clearly with peripheral nerve disease; he does not have a bullying or intimidating manner; his story is consistent; he doesn't have antisocial personality disorder. There is no evidence for a factitious disorder, either: He has no history of seeking the sick role, such as by repeated hospitalizations. In the presence of a well-documented general neurologic disorder—in this case, alcoholic peripheral neuropathy—one should not diagnose conversion disorder. In general, one should avoid diagnosing conversion disorder: It is better to say "such and such symptom or sign, etiology unknown." Although peripheral neuropathy commonly occurs along with Korsakoff's psychosis (Korsakoff's syndrome), there is no evidence of a severe memory deficit with confabulation in this case.

A 66-year-old right handed woman had abrupt onset of right face and hand weakness, impaired speech and language, and right homonymous hemianopia. This is most likely due to occlusion of which of the following arteries? A) Left middle cerebral B) Left anterior cerebral C) Left vertebrobasilar D) Right anterior choroidal E) Left posterior inferior cerebellar

21. (A) Left middle cerebral artery occlusion. This artery supplies the lateral convexity of the left cerebral hemisphere, including the frontal and temporal lobes. This explains the aphasia, the hemianopia and the right hand and face weakness. Right homonymous hemianopia results from a left hemisphere lesion anywhere along the left optic radiations, which run in the temporal lobes and the temporo-occipital regions all the way to the occipital lobe. Vertebro-basilar stroke presents with vertigo, diplopia, dysarthria, bifacial numbness, ataxia and weakness or numbness of part or all of one or both sides of the body. The anterior cerebral artery supplies the anterior three-quarters of the medial surface of the cerebral hemisphere, including the medial-orbital surface of the frontal lobe, the frontal pole, a strip of the lateral surface of the cerebral hemisphere along the superior border, and the anterior four fifths of the corpus callosum. Severe anterior cerebral artery occlusion often causes infarction of the anterior and medial parts of one or both cerebral hemispheres (depending on where the artery originates) resulting in hemiplegia or paraplegia, incontinence, motor aphasia (if the left hemisphere is affected) and frontal lobe personality changes. Transcortical motor aphasia may occur with occlusions of Heubner's branch of the left anterior cerebral artery. The anterior choroidal artery arises from the internal carotid artery and supplies the internal segment of the globus pallidus and posterior limb of the internal capsule, and then penetrates into the temporal horn of the lateral ventricle where it supplies the choroid plexus and anastomoses with the posterior choroidal artery. There is no classic anterior choroidal artery occlusion syndrome, but sometimes there is contralateral hemiplegia, hemi-hypesthesia and homonymous hemianopia due to involvement of the posterior limb of the internal capsule and white mater postero-lateral to it, through which the geniculocalcarine tract passes. With right sided lesions there may be left spatial neglect and constructional apraxia. The patient has none of these findings. Occlusion of the left posterior inferior cerebellar artery (PICA, lateral medullary syndrome) is associated with, a) if the vestibular nuclei are affected, nystagmus, nausea, vomiting and vertigo; b) if the inferior cerebellar peduncle is affected, ipsilateral cerebellar signs like ataxia, dysmetria and dysdiadochokinesis; c) if the nucleus ambiguous of cranial nerves 9-11, or the vagal nerve roots are affected, ipsilateral laryngeal, pharyngeal and palatal hemiparesis occur with loss of the gag reflex, dysarthria, dysphagia and dysphonia (hoarseness) occur; d) if the glossopharyngeal nerve roots are affected, the gag reflex is lost; e) if the spinothalamic tracts are affected, there is contralateral loss of pain and temperature sensation; f) if the spinal trigeminal nucleus and tract are affected, there is ipsilateral loss of pain and temperature sensation from the face; and g) if the descending sympathetic tract is affected, ipsilateral Horner's syndrome (ptosis, meiosis and anhidrosis) occurs. The patient has none of these findings.

For the past two months, a 57-year-old man has had fine postural tremors of his hands and head. He takes no medication that is associated with tremor. There is no past history of psychiatric illness, exposure to toxic substances, or clinically significant anxiety, and no evidence of thyroid disease. His mother and maternal grandfather had the same type of tremor which responded to medication. There is no family history of behavioral disorders. In a thorough examination, there are no abnormalities except for the tremor. Of the following, the best treatment of this condition would be A) A typical antipsychotic (neuroleptic) like haloperidol B) An atypical antipsychotic (neuroleptic) like olanzapine C) An anticholinergic agent like benztropine D) A benzodiazepine anxiolytic like lorazepam E) A beta adrenergic blocker like propranolol

1. (E) A beta adrenergic blocker like propranolol is the treatment of this benign essential tremor, which tends to be familial, sometimes autosomal dominant with variable penetrance. It tends to be absent during rest and to intensify with writing, drinking from a filled glass or supporting an envelope with an outstretched hand, and anxiety. Like most tremors, it tends to occur in a single plane; for example, when it affects the head, it tends to present as a "yes" (nodding up and down) or "no" (shaking head side to side) tremor. Typical and atypical antipsychotics (neuroleptics) are not used for treating tremor, and can (especially typical neuroleptics) cause parkinsonian tremor. Although benzodiazepines reduce anxiety, they are not a treatment for benign essential tremor. Anticholinergic agents may be effective for parkinsonian tremors, but not benign essential tremor. Other possible treatments of benign essential tremor include primidone (Mysoline), botulinum toxin (Botox) and deep brain stimulation.

A 58-year-old right handed college professor who had a severe closed head injury (Glasgow Coma Scale <10) following a fall two years ago is brought to see you by his wife. His wife is angry because he acts inappropriately in social situations. He often makes crude remarks, shows undue familiarity with strangers, and asks inappropriate personal questions. Her repeated attempts to explain this to the patient have been unsuccessful, and—because he is easily irritated—they often argue about it. Prior to the injury, his self control, interpersonal skill and manners were excellent, and they rarely fought. His post-injury behaviors most closely resemble those of which of the following primary psychiatric illnesses? A) Schizophrenia, paranoid type B) Schizoid personality disorder C) Mania D) Delusional disorder E) Dependent personality disorder

10. (C) Irritability, undue familiarity and intrusiveness are common in mania, but not schizophrenia (emotional blunting, first rank symptoms, formal thought disorder), delusional disorder (almost-believable delusions that tell a story like a movie plot), dependent personality disorder (excessive reliance on others) or schizoid personality disorder (social withdrawal, lack of close relationships, emotional coldness).

For 30 years, a 55-year-old woman has taken medication for schizoaffective disorder. She is alert. Frequently while she answers the interviewer's questions, she thrusts her tongue out of her mouth, makes chewing movements, and rolls her tongue against her cheek, and has non-rhythmic, writhing and jerky movements of her upper and lower extremities, torso and pelvis. A) Huntington's disease B) Obsessive compulsive disorder C) Parkinson's disease D) Progressive supranuclear palsy (we will not teach or test this disorder, although it could be tested on USMLE Step 1 or the NBME shelf exam) E) Sydenham's chorea F) Tardive dyskinesia G) Wilson's disease

10. (F) Tardive dyskinesia, characterized here by choreoathetoid movements resulting from long-standing use of neuroleptic medication.

A 10-year-old boy was neuropsychiatrically well until two weeks ago when he developed involuntary non-rhythmic writhing and jerky movements of his tongue, jaw, neck, trunk and pelvis. Some time in the past, he had a sore throat for which he received no laboratory tests or antibiotics. A) Huntington's disease B) Obsessive compulsive disorder C) Parkinson's disease D) Progressive supranuclear palsy (we will not teach or test this disorder, although it could be tested on USMLE Step 1 or the NBME shelf exam) E) Sydenham's chorea F) Tardive dyskinesia G) Wilson's disease

11. (E) Sydenham's chorea, characterized here by choreoathetoid movements developing after an untreated streptococcal sore throat.

A 66-year-old right-handed woman with atrial fibrillation is brought to the emergency room with right sided weakness and labored, slurred speech. Onset occurred suddenly 30 minutes ago while she was washing her face. Previously, she was neuropsychiatrically well. Examination reveals an irregularly irregular heartbeat, right upper extremity and right facial weakness, and slow, labored. dysarthric, telegraphic speech. Her blood pressure is normal, her skin has no petechiae, ecchymoses or hematomas, and her neck is supple. Which of the following is affected? A) Left frontal lobe B) Left arcuate fasciculus C) Left posterior superior temporal gyrus D) Right temporoparietal region E) Corpus callosum and left visual association cortex

17. (A) Right sided weakness and Broca's aphasia reveal a left frontal lesion. Arcuate fasciculus disease causes conduction aphasia (with normal fluency, spontaneity and prosody; paraphasic speech; impaired repetition; relatively normal comprehension and reading comprehension but inability to read out loud correctly); and right and left hand ideokinetic dyspraxia. Left posterior superior temporal gyrus disease causes Wernicke's aphasia (with normal fluency, spontaneity and prosody; impaired comprehension and repetition, and paraphasic speech). Right temporoparietal disease is associated with left sided sensory impairment; impaired prosodic comprehension; constructional and dressing dyspraxia; left hand kinesthetic dyspraxia; left hand stereoagnosia and left hand graphanesthesia. A combined lesion of the corpus callosum and left visual cortex is associated with alexia without agraphia.

A 66-year-old right-handed woman with atrial fibrillation is brought to the emergency room with right sided weakness and labored, slurred speech. Onset occurred suddenly 30 minutes ago while she was washing her face. Previously, she was neuropsychiatrically well. Examination reveals an irregularly irregular heartbeat, right upper extremity and right facial weakness, and slow, labored. dysarthric, telegraphic speech. Her blood pressure is normal, her skin has no petechiae, ecchymoses or hematomas, and her neck is supple. The next move the doctor should make is A) Observe the patient, unmedicated, for the next 24 hours to determine whether this is a transient ischemic attack or completed stroke. B) Obtain a cerebral arteriogram to ascertain whether the patient has an aneurysm or arterio-venous malformation. C) Order a CT scan of her head to rule out cerebral hemorrhage as the cause of her problem. D) Administer tissue plasminogen activator to reduce the extent of her cerebral ischemia. E) Telephone a family member to determine whether she has experienced spousal abuse or head trauma.

18. (C) Order a CT scan of her head to rule out cerebral hemorrhage as the cause of her problem, since CT identifies cerebral bleeding. With this clinical picture suggesting embolism, observing the patient unmedicated would be negligent. If cerebral bleeding is present, tissue plasminogen activator (TPA) is contraindicated. If not present, the next/second move would be to give TPA. The clinical picture of atrial fibrillation in the absence of profound headache or stiff neck makes ruptured berry aneurysm unlikely, so a cerebral arteriogram is not indicated at this point. It would be valuable to telephone a family member, but the clinical picture points more to embolism, and in the absence of visible bruising or skull fracture or the patient's giving a history of head trauma, head injury is not the main consideration.

A 77-year-old right handed married man, who was neuropsychiatrically well until three months ago, is admitted with headache, disorientation and left homonymous hemianopia. He recently had two brief episodes of unresponsiveness. He was never hypertensive. Alertness is seriously reduced. Temperature is normal. Mini-mental State score is 15/30. CT scan shows right parieto-occipital hemorrhage. 19. Of the following, the patient is most likely to have A) Normal pressure hydrocephalus B) Alzheimer's disease C) Fronto-temporal dementia D) Delirium E) Middle cerebral artery embolism

19. (D) Delirium, characterized in this case by severe reduction in alertness, disorientation and Mini-mental state score of 15. Normal-pressure hydrocephalus usually presents with the triad of gait abnormality, urinary incontinence and dementia, and not usually with headache and homonymous hemianopia. It tends to affect the frontal lobes rather than the temporo-parietal region. Alzheimer's disease is ruled out because it does not present with headache, homonymous hemianopia or parieto-occipital hemorrhage. Fronto-temporal dementia presents primarily with frontal and temporal lobe abnormalities, not parieto-occipital ones, and not with parieto-temporal hemorrhage. The middle cerebral artery supplies the lateral cortical convexity, primarily in the frontal and temporal, not parieto-occipital regions.

A 58-year-old married writer, with a 30-year history of heavy drinking with blackouts, morning drinking and gastritis, is brought by his wife to the hospital because of trouble standing and walking of four weeks' duration. He is cooperative, his history is credible and consistent, and he does not try to intimidate the staff. Currently he cannot stand or walk. Muscle strength in his feet and legs is severely reduced, calves are atrophic, ankle jerks are absent, position and vibration sense in his toes are impaired, plantar reflexes are flexor, and there is no ankle clonus. If he stops drinking, eats well, maintains social supports, and takes multivitamins, thiamine and physical therapy, his ability to walk will A) Not increase until he receives disability payments B) Not increase regardless of what treatment he receives C) Increase only if he receives antidepressants and psychodynamic psychotherapy D) Probably increase E) Deteriorate gradually and progressively, and he will die from respiratory failure

2. (D) His ability to walk will probably increase if he stops drinking and is properly treated with good diet, multivitamins including thiamine and physical therapy, and has good social supports.

For the past seven years, an 81-year-old man has had progressively increasing trouble walking, bowling, driving, carrying objects and, in the last two months, solving problems and swallowing. Because of the dysphagia, he chokes easily, must cut his food - or ask his wife to cut it - into tiny pieces, and has lost 15 pounds. Because of his gait problem, he has had a serious fall with a neck injury. He is cachectic. His facial muscles are rigid and expressionless. He has a bilateral rhythmic resting tremor with thumbs apposing forefingers. He needs to use a cane, and walks with slow, shuffling steps that sometimes speed up with a high risk of his falling. Of the following, the most likely diagnosis is A) Huntington's disease B) Alzheimer's disease C) Major neurocognitive disorder due to Lewy body disease (Lewy body dementia) D) Parkinson's disease E) Progressive supranuclear palsy (We will not teach or test this disorder, although it could be tested on USMLE Step 1 or the NBME shelf exam.)

2. (D) Parkinson's disease, characterized here by gradual and progressive neurologic dysfunction with rhythmic pill-rolling resting tremor; shuffling gait with festination and trouble walking with falls; expressionless face; and trouble with complex learned motor tasks like driving and bowling. Although late-age onset also typifies Alzheimer's disease, Alzheimer's does not typically present with tremor or problems with complex learned motor behaviors. Huntington's disease typically begins in early or middle adulthood, is strongly familial with autosomal dominant transmission, complete penetrance and a multigenerational history, and choreoathetoid movements. Dementia due to Lewy body disease (Lewy body dementia) also presents with parkinsonian features, along with fluctuating cognition, delusions and hallucinations (typically visual). Also common with it are an REM behavior disorder and adverse reactions to neuroleptics, which are contraindicated.

A nine-year-old boy has a two-year history of gradually progressive weakness of his shoulders, quadriceps, hip flexors and extensors. To arise from a prone position, he pushes himself up with his hands successively on the floor, then his knees and then his thighs. Two males in his family had a similar disorder. His shoulders and calves are very large but not strong. Of the following, the most likely diagnosis is A) Myasthenia gravis B) Guillain-Barre syndrome C) Duchenne's muscular dystrophy D) Charcot-Marie-Tooth syndrome E) Hypothyroid myopathy

3. (C) Duchenne's muscular dystrophy, characterized in this case by childhood onset of proximal muscle weakness; using Gower's maneuver (arising by using one's arms to push up against the floor and then "climbing one's own body" by grasping and pushing up on one's knees and thighs); enlarged but very weak shoulders and calves (pseudohypertrophy); and a family history suggesting X-linked genetic transmission. Muscle biopsy would show little or no dystrophin, and blood testing would show DNA deletions on the X chromosome. Myasthenia gravis, caused by acetylcholine receptor autoantibodies, typically presents with weakness of extraocular, facial and bulbar muscles, most prominent with repeated use of these muscles, typically at the end of the day. It most often affects young women or older men. Guillain-Barré syndrome, an acute imflammatory or post-infectious demyelinating polyneuropathy most common in young or middle-aged adults, typically begins with numbness and paresthesias of the fingers and toes, followed by areflexic, flaccid paralysis of the feet and legs. Weakness, which becomes a much greater problem than numbness, ascends to involve the hands and arms. Many patients develop respiratory distress from involvement of the phrenic and intercostal nerves and must be intubated for respirator assistance. If weakness ascends further, patients develop cranial nerve involvement that leads to dysphagia and other types of bulbar palsy, facial weakness and ocular immobility. Because optic and acoustic nerves are protected by myelin generated by the CNS—not the peripheral nervous system—patient continue to see and hear. Also noteworthy is that bladder, bowel and sexual function are preserved, typical of neuropathies except for diabetic neuropathy. The CSF shows albuminocytologic dissociation, with elevated protein but few white cells. The illness usually resolves completely within 3 weeks to 3 months as the peripheral nervous system myelin is regenerated. Charcot-Marie-Tooth syndrome is one of several polyneuropathies characterized by genetic transmission, complete symmetry, slow progression, and axon-myelin fiber loss. It is most often autosomal dominant with complete penetrance, but can also be X-linked dominant or X-linked recessive. Most cases result from duplication of the gene for a peripheral myelin protein on chromosome 17. Most often, Charcot-Marie-Tooth disease begins during late childhood or adolescence. In younger children, the child has difficulty running, weakness and sprains of the ankles, or stumbling. There is chronic degeneration of peripheral nerves and roots with distal muscle atrophy beginning in the feet and legs and later involving the hands. Later, all leg muscles and sometimes the lower third of the thighs become weak and atrophic. The thin legs have been likened to those of a stork (stork legs) or, if the lower thigh muscles are affected, to an "inverted champagne bottle." Genetic testing is available. Hyperthyroid myopathy is characterized by progressive weakness and wasting of skeletal musculature, most prominent in the pelvic girdle and thighs, usually of mild to moderate severity, associated with hyperthyroidism. With proper treatment of the thyrotoxicosis, muscle power and mass gradually improve.

A 45-year-old nursing home resident just recovered from an atypical, non-melancholic depression with suicidal ideation that lasted four weeks. He was neuropsychiatrically well until six years ago, when he started having writhing and jerky movements of his face, extremities and trunk. At age 40, he was treated for an atypical depression. He never had a manic or hypomanic episode. His mother is demented and has a movement disorder. His maternal grandfather and great grandfather died in state psychiatric hospitals. His alertness and mood are normal. He has jerky non-rhythmic movements of his face and extremities, and this makes his gait slow and awkward. There is no tremor, hepatomegaly or corneal abnormalities. Mini-mental State score is 22/30. Of the following, the most likely diagnosis is A) Major neurocognitive disorder due to vascular disease (Vascular dementia) B) Major neurocognitive disorder due to familial Alzheimer's disease (Familial Alzheimer's disease) C) Major neurocognitive disorder due to Huntington's disease (Dementia due to Huntington's disease) D) Bipolar II disorder E) Major neurocognitive disorder due to Wilson's disease (Dementia due to Wilson's disease)

3. (C) Major neurocognitive disorder due to Huntington's disease (Dementia due to Huntington's disease), characterized here by onset at 39, a multigenerational history, choreoathetoid movements, an atypical mood disorder, and impaired cognition. Major neurocognitive disorder due to vascular disease (Vascular dementia) tends to a later age of onset, risk factors for stroke, and stepwise illness progression. In major neurocognitive disorder due to vascular disease, choreoathetosis is rare. Major neurocognitive disorder due to Alzheimer's disease (Dementia due to Alzheimer's disease) is characterized by late age of onset and temporoparietal dysfunction without movement disorder. Bipolar disorder is not associated with choreoathetosis; depression in bipolar disorder is typical depression, often melancholic. Hepato-lenticular degeneration (Wilson's disease) tends to present at earlier ages than Huntington's disease, is more likely to be associated with flapping hand tremor or wing-beating shoulder tremor than choreoathetosis (although choreoathetosis can occur), and is associated with cirrhosis and Kayser-Fleischer corneal rings.

A 45-year-old nursing home resident just recovered from an atypical, non-melancholic depression with suicidal ideation that lasted four weeks. He was neuropsychiatrically well until six years ago, when he started having writhing and jerky movements of his face, extremities and trunk. At age 40, he was treated for an atypical depression. He never had a manic or hypomanic episode. His mother is demented and has a movement disorder. His maternal grandfather and great grandfather died in state psychiatric hospitals. His alertness and mood are normal. He has jerky non-rhythmic movements of his face and extremities, and this makes his gait slow and awkward. There is no tremor, hepatomegaly or corneal abnormalities. Mini-mental State score is 22/30. Which of the following brain imaging findings is most likely? A) Caudate nucleus atrophy bilaterally B) Temporo-parietal hypometabolism C) Calcification of the globus pallidus and putamen D) Patchy demyelination in multiple structures E) Large hematoma encroaching on his non-dominant parietal lobe

4. (A) Caudate nucleus atrophy bilaterally. Temporoparietal hypometabolism typifies Alzheimer's disease. Caudate and putamen calcification occurs in Fahr's disease (idiopathic basal ganglia calcification), which typically occurs between ages 20 and 40 and manifests a schizophrenia-like psychosis which eventually develops into a major neurocognitive disorder (dementia). Choreoathetosis and pyramidal tract signs may occur. Parathyroid and thyroid disease can also cause basal ganglia calcification. Patchy demyelination in multiple CNS structures typifies multiple sclerosis. A large hematoma encroaching on the parietal lobe can occur with subdural, epidural or intracerebral hematoma.

A 44-year-old man comes to his local clinic in rural India because of circumscribed lesions on his face, ears, forearms and calves. The edges of each lesion are inflamed and irregularly elevated, and the lesions' healed centers are normally-colored and anesthetic. The area under his wristband is unaffected. Deep tendon reflexes are normal. Mini-mental State score is 29/30. 4. Of the following, the most likely cause of this disorder is a A) Mycobacterium B) Heavy metal C) Spirochete D) Factitious disorder E) Herpes infection

4. (A) Mycobacterial infection, specifically infectious neuritis caused by direct invasion of nerves by the acid-fast Mycobacterium leprae. This is common in India and Central Africa, and also occurs in parts of South America and the U.S. Gulf Coast—Florida, Texas and Louisiana. The typical initial lesion is anesthetic and normally-colored in the centers, and inflamed and irregularly elevated on the edges. This is caused by invasion of the cutaneous nerves. The organism can spread to large nerves and hematogenously. Cool portions of the body, such as the nose, ear lobes and digits are most severely affected. Treatment is with sulfones, rifampin and clofazimine. The skin lesions respond to thalidomide, which may cause serious side effects including peripheral neuropathy and severe teratogenicity. Heavy metal toxicity includes toxicity from lead, mercury, iron, copper, manganese, cadmium, arsenic, nickel, aluminum, silver and beryllium. Lead neuropathy is uncommon. In adults, it follows chronic exposure to lead paint or fumes (as occurs in smelting or from burning batteries) and ingestion of liquor distilled in lead pipes. Its most characteristic clinical feature is the predominantly motor affection, mainly in the distribution of the radial nerves (wrist and finger drop). Less commonly, foot drop is observed. Arsenic is used in many insecticide sprays, as a disinfectant in animal hides and furs, and in the manufacture of paints, prints and enamels, but its main toxicity is as a suicidal or homicidal agent. Chronic ingestion results in dementia, myelopathy, optic neuropathy and peripheral neuropathy. The peripheral neuropathy from chronic ingestion can resemble nutritional/alcoholic neuropathy. Acute ingestion can lead, after 8-21 days, to a Guillain-Barré-like syndrome. The dementia is associated with punctate hemorrhages in the white matter. There may be a metallic taste, transverse white lines (Mees' lines) on the nails, brownish pigmentation of the skin, hyperkeratosis of the palms and soles, and anemia. Laboratory diagnosis depends on arsenic concentrations above 0.1 mg/100g in the hair or nails. Mercury intoxication occurs during the manufacture of paper or thermometers, in the preparation of chlorine, from ingestion of contaminated seafood or grain containing methylmercury, from excessive use of calomel laxatives, or from breathing metallic mercury vapor. Inorganic mercury intoxication causes stomatitis, irritability in response to stimulation (erethism), tremor and peripheral neuropathy, but little cognitive impairment. Organic mercury intoxication causes paresthesias, ataxia, visual field constriction and blindness, deafness, dysarthria, choreoathetosis and an amyotrophic lateral sclerosis-like syndrome. Factitious disorder is ruled out because there is no evidence the patient habitually seeks to be in the sick role, such as by repeated hospitalizations. Herpes zoster (shingles) infections are characterized by painful erythematosus vesicular lesions at a single dermatome site.

A 44-year-old man comes to his local clinic in rural India because of circumscribed lesions on his face, ears, forearms and calves. The edges of each lesion are inflamed and irregularly elevated, and the lesions' healed centers are normally-colored and anesthetic. The area under his wristband is unaffected. Deep tendon reflexes are normal. Mini-mental State score is 29/30. This disorder is A) Highly prevalent in the United States and Europe B) More prevalent in cold than warm climates C) The world's most common neuropathy D) Unresponsive to treatment E) More frequent in homosexuals than heterosexuals

5. (C) Leprosy is the world's most common neuropathy. It is highly prevalent in India and Central Africa, and occurs occasionally in Gulf Coast states like Florida, Louisiana and Texas. It occurs most commonly in warm climates, has no relationship to sexual preference, and responds to treatment with sulfones.

A 70-year-old married man was neuropsychiatrically well until six years ago, when he started having tremors, gait problems and falling, which have increased gradually and progressively. Four months ago, he started having a sad mood with pessimism, apathy, trouble falling asleep, and occasional suicidal ideation without suicidal intent or plans. Recently, he became unable to play golf. He has no past or family history of psychiatric illness. He is alert, has an expressionless face, a slow gait with reduced arm swing, and muscle rigidity. He has a bilateral tremor, with his thumbs rhythmically apposing his forefingers, most prominent when his hands rest at his sides. His mood is sad, but it improves as the interview progresses. He has mild problems with recent and remote memory, and his memory improves with cuing. His Mini-mental State score is 24/30. Of the following, his mood problem is most likely due to A) Bilateral temporoparietal lobe abnormality B) Frontal-subcortical circuit disease C) A primary (classic, idiopathic) mood disorder D) Reduced cerebrospinal fluid absorption E) Catatonia due to bilateral brainstem dysfunction

6. (B) Parkinson's disease is a frontal-subcortical circuit disease. Bilateral temporoparietal dysfunction typifies Alzheimer's disease. The mood disorder described here is atypical, not what you would expect with a classic primary mood disorder. Reduced cerebrospinal fluid absorption is characteristic of normal-pressure hydrocephalus, not Parkinson's disease. Catatonia is not a typical Parkinson's disease finding.

A 45-year-old barber is hospitalized for "confusion and incoordination." He was neuropsychiatrically well until six weeks ago. Eight weeks ago, he began an extramarital affair. His wife found out about the affair and was furious at him. She told a close friend, "I'm so angry at that rat I could poison him." Six weeks ago the patient developed nausea, vomiting and trouble walking. Recently he has had severely reduced alertness and concentration. There is no urinary incontinence. His skin and conjunctivae are pale and yellow, his nails have transverse lines, ankle jerks are absent, Mini-mental State score is 16/30, and he is unable to respond to an "A" test. There is no ankle clonus, and plantar reflexes are flexor. 6. In addition to hemolytic anemia and jaundice he also has A) Alzheimer's disease B) Parkinson's disease C) Delirium D) Lewy body dementia E) Normal pressure hydrocephalus

6. (C) Delirium, characterized in this case by reduced alertness and concentration, low Mini-mental State score, nausea, vomiting, anemia with pallor, jaundice, transverse lines (Mees' lines) on the nails, peripheral neuropathy with trouble walking, and no signs of upper motor disease. Alzheimer's disease is rare at age 45; is gradually progressive over multiple years; and is not associated with motor abnormalities, anemia or jaundice. Parkinson's disease is also rare at age 45, is gradually progressive, and not associated with jaundice, anemia, nausea or vomiting. Lewy body dementia is rare at age 45, is gradually progressive over multiple years, and is characterized by parkinsonism, dementia, cognitive fluctuations and visual hallucinations. It is not associated with jaundice, anemia, nausea or vomiting. Normal-pressure hydrocephalus is associated with dementia with frontal lobe findings, gait abnormality, and urinary incontinence, but not with nausea, vomiting, anemia, jaundice or Mees' lines.

A 60-year-old man was neuropsychiatrically well until two months ago when he began having frequent falls which preceded onset of progressively increasing problems with swallowing, and "slowing down" of his gait. He has cogwheel rigidity of his axial muscles more than in his arms and legs. Although he can look down passively, he cannot do this voluntarily. A) Huntington's disease B) Obsessive compulsive disorder C) Parkinson's disease D) Progressive supranuclear palsy (we will not teach or test this disorder, although it could be tested on USMLE Step 1 or the NBME shelf exam) E) Sydenham's chorea F) Tardive dyskinesia G) Wilson's disease

7. (D) Progressive supranuclear palsy, characterized in this case by axial but not extremity rigidity; inability to look down; and dysphagia. It is more frequent in men than women, has a median age of onset of 63, and a median survival of 6 years. It is associated with an abnormality on chromosome 17 responsible for coding tau protein. Patients have atrophy of the upper brainstem, unusual neurofibrillary tangles, and abnormal tau deposition. We will not teach or test this disorder, although it could be tested on USMLE Step 1 or the NBME shelf exam. Obsessive compulsive disorder can result from basal ganglia disease, such as Wilson's disease, Huntington's disease, Sydenham's chorea, Parkinson's disease and Hallovorden-Spatz' disease. Parkinson's disease, as you know, is characterized by bradykinesia, rigidity, expressionless face, pill-rolling resting tremor, shuffling and festinating gait with reduced arm swing and proneness to falls, atypical depression or obsessive compulsive disorder and frontal-subcortical dementia.

A 45-year-old barber is hospitalized for "confusion and incoordination." He was neuropsychiatrically well until six weeks ago. Eight weeks ago, he began an extramarital affair. His wife found out about the affair and was furious at him. She told a close friend, "I'm so angry at that rat I could poison him." Six weeks ago the patient developed nausea, vomiting and trouble walking. Recently he has had severely reduced alertness and concentration. There is no urinary incontinence. His skin and conjunctivae are pale and yellow, his nails have transverse lines, ankle jerks are absent, Mini-mental State score is 16/30, and he is unable to respond to an "A" test. There is no ankle clonus, and plantar reflexes are flexor. Which of the following is most likely to determine the etiology of his condition? A) Complete blood count and chest x-ray B) Wechsler Adult Intelligence Scale C) Angiography of neck and brain vessels D) Serum creatine phosphokinase and porphobilinogen E) Hair clippings and 24-hour urinanalysis

7. (E) Hair clippings and 24-hour urinalysis yield the diagnosis of arsenic intoxication. Complete blood count would reveal anemia, but wouldn't be specific for arsenic toxicity. Chest x-ray would be unhelpful. The Wechsler Adult Intelligence Scale would be inappropriate for a delirious person, who wouldn't be alert enough for testing. Angiography of neck and brain vessels would miss arsenic toxicity. Serum CPK is valuable for identifying muscle disease or dysfunction, not arsenic toxicity. Porphobilinogen is present in the urine in porphyria, but its measurement is unhelpful for diagnosing arsenic toxicity.

A 45-year-old woman was neuropsychiatrically well until three months ago when she developed irritability, moodiness and atypical obsessive-compulsive symptoms. She has not been medically treated for this problem. She makes abrupt, rapid, unsustained, jerky non-rhythmic movements that seem to flow from one body part to another. MRI reveals a "butterfly" configuration of her lateral ventricles, caused by atrophy of a structure adjacent to each ventricle. A) Huntington's disease B) Obsessive compulsive disorder C) Parkinson's disease D) Progressive supranuclear palsy (we will not teach or test this disorder, although it could be tested on USMLE Step 1 or the NBME shelf exam) E) Sydenham's chorea F) Tardive dyskinesia G) Wilson's disease

8. (A) Huntington's disease, characterized in this case by atypical obsessive compulsive symptoms, choreiform movements, and caudate atrophy with a butterfly appearance of the lateral ventricles.

A 54-year-old man was neuropsychiatrically well until four months ago when he started having constant burning pain and paresthesias in his feet. When he sleeps, the pain often awakens him, and he prefers to uncover his feet because the sheet makes his shins uncomfortable. He has had diabetes mellitus for eight years. He drinks a glass of wine occasionally with dinner, and there is no history of morning drinking, blackouts, binges, tremor, guilt about drinking, or trouble with the law due to drinking. His behavioral brain examination reveals no abnormalities. Vibration and pain sensation are reduced in both feet and ankles, and ankle jerks are decreased bilaterally. 8. Of the following, the most likely diagnosis is A) Herpes zoster infection B) Multiple sclerosis C) Myopathy D) Peripheral neuropathy E) Alcohol abuse

8. (D) Peripheral neuropathy, specifically chronic progressive distal diabetic polyneuropathy, the most common type of diabetic neuropathy. It is a mixed sensory-autonomic-motor polyneuropathy—predominantly sensory. In about 10% of patients, pain is the most prominent feature. The sensory symptoms begin distally in the toes extending to the forelegs before involving the upper limbs. Most patients exhibit evidence of both small and large fiber involvement. About 15% of patients with diabetes mellitus have both symptoms and signs of some type of neuropathy, but nearly 50% have neuropathic symptoms or nerve conduction abnormalities. Neuropathy is most common in diabetics over 50, uncommon under 30 and rare in childhood. It is similar in frequency for type 1 and 2 diabetics. There are other types of diabetic neuropathy, including (1) acute diabetic ophthalmoplegia that affects the third nerve and less often the sixth nerve; (2) acute mononeuropathy of limbs or trunk including a painful thoracolumbar radiculopathy; (3) a rapidly evolving, painful, asymmetrical, predominantly motor multiple neuropathy affecting the upper lumbar roots and the proximal leg muscles and usually undergoing remission ("diabetic amyotrophy"); (4) a more symmetrical, proximal motor weakness and wasting, often without pain and with variable sensory loss that pursues a subacute or chronic course; (5) an autonomic neuropathy involving bowel, bladder and circulatory reflexes; and (6) a painful thoracoabdominal radiculopathy. These forms of neuropathy often co-exist and overlap. Herpes zoster is characterized by a painful red vesicular eruption over a single dermatome. In most myopathies, the large proximal muscles like the shoulder and hip girdle muscles are affected initially and most severely. In this patient, there is no evidence of alcohol abuse.

A 30-year-old woman has developed atypical depressive symptoms, dyskinesias, ataxia, corneal and hepatic abnormalities. A) Huntington's disease B) Obsessive compulsive disorder C) Parkinson's disease D) Progressive supranuclear palsy (we will not teach or test this disorder, although it could be tested on USMLE Step 1 or the NBME shelf exam) E) Sydenham's chorea F) Tardive dyskinesia G) Wilson's disease

9. (G) Wilson's disease, characterized in this case by onset age 30, choreoathetoid movements, corneal and hepatic abnormalities, and atypical depression.

A 66-year-old man was neuropsychiatrically well until three days ago when he suddenly experienced slurred speech and right upper extremity paralysis and numbness. There is no history of head injury. Heart rate is 102 and irregularly irregular. His speech is slow, labored and dysarthric, with impaired repetition and writing and relatively good comprehension. His right upper extremity is weak and hyperreflexic. Cerebrospinal fluid is clear and colorless. A) Cerebral embolus with infarction B) Epidural hematoma C) Intracerebral hemorrhage D) Ruptured saccular aneurysm E) Subdural hematoma

A) Cerebral embolus with infarction, based on the sudden onset of cognitive problems in a patient with atrial fibrillation.

A 25-year-old man was beaten by a man with a baseball bat. He does not speak or make any sounds. He opens his eyes to painful stimulation only. He shows a flexion response to pinching his forearms. CT scan shows bilateral cortical contusions. 7. His Glasgow Coma Scale (GCS) score is A) 0 B) 4 C) 6 D) 11 E) 15

B

While playing football, a 14-year-old boy is tackled hard and knocked unconscious for 10 seconds. Immediately afterwards, because of his symptoms, two of his friends accompany him home. He has mild dizziness, bilateral fronto-temporal headache, photophobia, and mild problems paying attention, which concern his parents who take him to the local emergency room. In the emergency room physical exam, the only abnormality is mild trouble with concentration. CT scan reveals no abnormality, but the EEG shows mild non-specific focal slowing. The headache and dizziness persist for 12 hours after he leaves the emergency room, and then subside gradually over two hours. Of the following, which is the most likely diagnosis? A) Frontal lobe subdural hematoma B) Post-concussive disorder C) Orbito-frontal syndrome D) Subarachnoid bleeding due to ruptured berry aneurysm E) Malingering to avoid having to take an upcoming test

B) Post-concussive disorder, based on the history of brief unconsciousness (<30 minutes), and the findings of photophobia, bilateral headache, dizziness, poor concentration, and mild non-specific EEG slowing. Acute or subacute subdural hematoma often presents with delirium or coma, either immediately or after a lucid interval, and sometimes presents with pupillary dilatation or hemiparesis. If the frontal lobe is injured, frontal signs can appear, including dysexecutive syndrome or orbitomedial disinhibited syndrome. Subarachnoid bleeds due to ruptured berry aneurysm typically produce sudden profound headache, nuchal rigidity, nausea, delirium and, sometimes, focal neurologic dysfunctions. Orbitofrontal syndrome is characterized by irritability, impulsivity, silly fatuous euphoria (Witzelsucht), coarsening of the personality, utilization behavior or intrusiveness. There is no evidence for malingering: The boy has no evidence of conduct disorder or substance abuse, does not give a conflicting history or try to intimidate the examiner, and there is no obvious legal or monetary problem in the background.

A 77-year-old right handed married man, who was neuropsychiatrically well until three months ago, is admitted with headache, disorientation and left homonymous hemianopia. He recently had two brief episodes of unresponsiveness. He was never hypertensive. Alertness is seriously reduced. Temperature is normal. Mini-mental State score is 15/30. CT scan shows right parieto-occipital hemorrhage. Of the following, the most likely cause of his condition is A) Atrial fibrillation B) Cocaine intoxication C) Amyloid angiopathy D) Unrecognized splenic rupture E) Diplococcal meningitis

C) Amyloid angiopathy is a common cause of intracerebral hemorrhage. Atrial fibrillation is associated with an irregularly irregular pulse and tends to produce embolization with vascular occlusion. Absent a prior history of drug abuse, hypertension or tachycardia, cocaine toxicity is extremely unlikely in a 77-year-old married man. Cocaine blocks pre-synaptic reuptake of biogenic amines, produces vasoconstriction, hypertension, tachycardia and euphoria; and is more apt to produce vascular occlusion and infarction than hemorrhage (which may occur). Splenic rupture does not cause intracerebral hematoma, and there is no evidence in this case of shock or anemia. Diplococcal meningitis presents with fever and stiff neck, neither of which is present here.

While touring the Empire State Building in New York City, a 23-year-old rock band leader was shot in the head by a psychotic man. Right after being shot, he became comatose. The gunshot wound required surgical evacuation of an intracranial hematoma. After recovering from the surgery, he could no longer make plans for the band, or compose songs. Before the injury, he was socially skillful, but afterwards he lost some of his social graces and became intrusive. Two years later he took a job selling CDs at a store, but was fired because he told customers that they could find better products elsewhere. His recognition of faces, semantic memory, episodic memory, visual acuity, reading and writing ability are good. Which of the following brain structures was most severely injured by the gunshot? A) Occipital lobes bilaterally B) Cerebellar vermis C) Frontal lobes D) Left temporo-parietal region E) Right temporo-parietal region

C) Frontal lobes, based on the findings of impaired planning, song composition, social graces and judgment, in the presence of normal facial recognition, semantic and episodic memory, visual acuity, reading and writing. Bilateral occipital lobe injury could cause bilateral blindness, visual agnosia, unilateral or bilateral achromatopsia and unilateral or bilateral Balint syndrome. Cerebellar vermis injury causes truncal ataxia. Left temporoparietal injury causes Wernicke's or conduction aphasia; bilateral ideokinetic dyspraxia; right hand kinesthetic dyspraxia, stereoagnosia and graphanesthesia; and Gerstmann's syndrome (acalculia, agraphia, left-right disorientation and finger agnosia). Right temporoparietal injury causes impaired comprehension of others' prosody; dressing dyspraxia; left spatial neglect; constructional dyspraxia; left hand kinesthetic dyspraxia, graphanesthesia and stereoagnosia; and prosopagnosia with Capgras or Fregoli syndrome.

A 25-year-old man was beaten by a man with a baseball bat. He does not speak or make any sounds. He opens his eyes to painful stimulation only. He shows a flexion response to pinching his forearms. CT scan shows bilateral cortical contusions. Based on his GCS score, the statistical likelihood of his dying from this injury is about A) 1% B) 4% C) 10% D) 25% E) 40%

E

A 24-year-old Army tank driver leaned on his elbows for dozens of hours. For the past two months, he has had numbness and tingling in the little and ring fingers bilaterally, and bilateral wasting of the thumb adductors and first dorsal interosseous muscles. A) Axillary nerve neuropathy B) Median nerve neuropathy C) Muscular dystrophy D) Radial nerve neuropathy E) Ulnar nerve neuropathy

E) Ulnar nerve neuropathy, characterized by numbness and paresthesias in the little and ring fingers bilaterally, and bilateral wasting of the thumb adductors and first dorsal interosseus muscles, all innervated by the ulnar nerve.

Doctor: Tell me what you see in this picture. Patient: It could have done it, if I took to Joe there were not any. Doctor: Please, tell me what you see in this picture. Patient: I like A, B or C, but don't give me five or seven. Doctor: Do you understand what I'm telling you? Patient: Big sevens I don't do well. Doctor: Repeat after me, say "no ifs ands or buts." Patient: That's why they took so many of them. A) Broca's aphasia B) Conduction aphasia C) Extrasylvian (transcortical) sensory aphasia D) Extrasylvian (transcortical) motor aphasia E) Global aphasia F) Pure word deafness G) Wernicke's aphasia

G) Wernicke's aphasia, characterized by normal spontaneity and fluency, impaired comprehension and repetition, and paraphasic speech. Broca's aphasia is characterized by slow, labored, dysarthric telegraphic speech, with impaired repetition, slightly impaired comprehension for sentences with many functor words, and no paraphasic speech. Conduction aphasia shows normal spontaneity, fluency and prosody, impaired repetition, paraphasic speech, and normal comprehension and reading comprehension but paraphasic speech when reading out loud. Extrasylvian (transcortical) sensory aphasia includes normal spontaneity and fluency, impaired comprehension, paraphasic speech, and normal repetition with—often—echolalia. Global aphasia is typified by a combination of Broca's and Wernicke's aphasia, with slow, labored, dysarthric telegraphic speech; impaired comprehension and repetition; and paraphasic speech. Pure word deafness is characterized by inability to comprehend others' speech due to deafness, with high pitched speech, normal spontaneity, and no paraphasic speech.

Doctor: Tell me what you see in this picture. Patient: They like very much of them, it is good to do that. Doctor: Please, tell me what you see in the picture. Patient: It is the uncle who gives 11 of them. Doctor: Do you understand what I'm saying? Patient: Six of one, two dozen are not best. Doctor: Repeat after me, say 'No ifs ands or buts.' Patient: B or A, don't try it with them. A) Broca's aphasia B) Catatonia C) Circumstantiality D) Extrasylvian sensory aphasia E) Normal language, speech and cognitive function, no abnormality present F) Spatial neglect G) Wernicke's aphasia

G) Wernicke's aphasia, characterized by normal spontaneity and fluency, impaired repetition and comprehension, and paraphasic speech. There are no signs of catatonia (e.g., echolalia, verbigeration, automatic obedience) in this example. Circumstantiality is characterized by excessive use of detail, most common in mania, alcoholism, epilepsy, aging and stimulant intoxication). None of these language samples reveals normal speech and language. There is no evidence of spatial neglect in any of these samples.


Ensembles d'études connexes

Phlebotomy Essentials 6th edition. ALL quizzes, ALL ch. tests, GRADED work, NOT guesses. PLUS, the FULL NAHP study guide

View Set

MICRO mastering vaccines and immunological testing

View Set

7th grade S.S. Daily Geo from 11/3

View Set

10. Processes and System Services

View Set

Geometry 1.11: Use Algebra to Describe Geometry 2

View Set

Dynamic Routing Protocol Homework

View Set